OB Exam 2 joscircus

¡Supera tus tareas y exámenes ahora con Quizwiz!

Intervention to reduce preterm birth can be divided into primary prevention and secondary prevention. What does secondary prevention include? 1. Diagnosis and treatment of infections 2. Cervical cerclage 3. Progesterone administration 4. Antibiotic treatment and tocolysis

Answer: 4 Explanation: 4. Secondary prevention strategies are antibiotic treatment and tocolysis.

The charge nurse is looking at the charts of laboring clients. Which client is in greatest need of further intervention? 1. Woman at 7 cm, fetal heart tones auscultated every 90 minutes 2. Woman at 10 cm and pushing, external fetal monitor applied 3. Woman with meconium-stained fluid, internal fetal scalp electrode in use 4. Woman in preterm labor, external monitor in place

: 1 Explanation: 1. During active labor, the fetal heart tones should be auscultated every 30 minutes; every 90 minutes is not frequent enough.

Which of the following is a common barbiturate used in labor? 1. Seconal 2. Valium 3. Phenergan 4. Vistaril

1. Seconal

To reduce possible side effects from a cesarean section under general anesthesia, clients are routinely given which type of medication? 1. Antacids 2. Tranquilizers 3. Antihypertensives 4. Anticonvulsants

Answer: 1 Explanation: 1. Antacids are routinely administered before surgery for a cesarean section.

What would be a normal cervical dilatation rate in a first-time mother ("primip")? 1. 1.5 cm per hour 2. Less than 1 cm cervical dilatation per hour 3. 1 cm per hour 4. Less than 0.5 cm per hour

Answer: 1 Explanation: 1. Dilatation in a "multip" is about 1.5 cm per hour.

Which of the following conditions would predispose a client for thrombophlebitis? 1. Severe anemia 2. Cesarean delivery 3. Anorexia 4. Hypocoagulability

Answer: 1 Explanation: 1. Severe anemia would predispose a client for thrombophlebitis.

Clinical features of posttraumatic stress disorder (PTSD) include which of the following? 1. Difficulty sleeping 2. Acute awareness 3. Flashbacks 4. The need to be constantly around others 5. Irritability

Answer: 1, 3, 5 Explanation: 1. A clinical feature of PTSD is difficulty thinking. 3. A clinical feature of PTSD is intrusive thoughts and flashbacks to the threatening event. 5. A clinical feature of PTSD is irritability.

To prevent the spread of infection, the nurse teaches the postpartum client to do which of the following? 1. Address pain early 2. Change peri-pads frequently 3. Avoid overhydration 4. Report symptoms of uterine cramping

Answer: 2 Explanation: 2. Changing peri-pads frequently decreases skin contact with a moist medium that favors bacteria growth.

The nurse has just palpated a laboring woman's contractions. The uterus cannot be indented during a contraction. What would the intensity of these contractions best be characterized as? 1. Weak 2. Mild 3. Moderate 4. Strong

Answer: 4 Explanation: 4. Strong intensity exists when the uterine wall cannot be indented.

The nurse is completing discharge teaching for a client who delivered 2 days ago. Which statement by the client indicates that further information is required? 1. "Because I have a midline episiotomy, I should keep my perineum clean." 2. "I can use an ice pack to relieve some the pain from the episiotomy." 3. "I can take ibuprofen (Motrin) when my perineum starts to hurt." 4. "The tear I have through my rectum is unrelated to my episiotomy."

Answer: 4 Explanation: 4. This statement is incorrect. The major disadvantage is that a tear of the midline incision may extend through the anal sphincter and rectum.

On assessment, a labor client is noted to have cardiovascular and respiratory collapse and is unresponsive. What should the nurse suspect? 1. An amniotic fluid embolus 2. Placental abruption 3. Placenta accreta 4. Retained placenta

answer 1 Explanation: 1. Cardiovascular and respiratory collapse are symptoms of an amniotic fluid embolus and cor pulmonale.

The nurse is making client assignments for the next shift. Which client is most likely to experience a complicated labor pattern? 1. 34-year-old woman at 39 weeks' gestation with a large-for-gestational-age (LGA) fetus 2. 22-year-old woman at 23 weeks' gestation with ruptured membranes 3. 30-year-old woman at 41 weeks' gestation and estimated fetal weight 7 pounds 8 ounces 4. 43-year-old woman at 37 weeks' gestation with hypertension

Answer: 1 Explanation: 1. A risk factor for hypotonic uterine contraction patterns includes a large-for-gestational-age (LGA) fetus.

How would the nurse best analyze the results from a client's sonogram that shows the fetal shoulder as the presenting part? 1. Breech, transverse 2. Breech, longitudinal 3. Breech, frank 4. Vertex, transverse

Answer: 1 Explanation: 1. A shoulder presentation is one type of breech presentation, and is also called a transverse lie.

A laboring client's obstetrician has suggested amniotomy as a method for creating stronger contractions and facilitating birth. The client asks, "What are the advantages of doing this?" What should the nurse cite in response? 1. Contractions elicited are similar to those of spontaneous labor. 2. Amniotomy decreases the chances of a prolapsed cord. 3. Amniotomy reduces the pain of labor and makes it easier to manage. 4. The client will not need an episiotomy.

Answer: 1 Explanation: 1. Contractions after amniotomy are similar to those of spontaneous labor.

The nurse would expect a physician to prescribe which medication to a postpartum client with heavy bleeding and a boggy uterus? 1. Methylergonovine maleate (Methergine) 2. Rh immune globulin (RhoGAM) 3. Terbutaline (Brethine) 4. Docusate (Colace)

Answer: 1 Explanation: 1. Methylergonovine maleate is the drug used for the prevention and control of postpartum hemorrhage.

A woman in active labor is given nalbuphine hydrochloride (Nubain) 14 mg IV for pain relief. Half an hour later, her respirations are at 8 per minute. The physician would likely order which medication for this client? 1. Narcan 2. Reglan 3. Benadryl 4. Vistaril

Answer: 1 Explanation: 1. Narcan is useful for respiratory depression caused by nalbuphine (Nubain).

A woman who is 40 weeks pregnant calls the labor suite to ask whether she should be evaluated. Which statements by the client indicate she is likely in labor? Note: Credit will be given only for all correct choices and no incorrect choices. Select all that apply. 1. "The contractions are 5-20 minutes apart." 2. "I had pink discharge on the toilet paper." 3. "I have had cramping for the past 3-4 hours." 4. "The contractions start in my back and then go to my abdomen and are very intense." 5. "The contractions hurt more when I walk."

Answer: 4, 5 Explanation: 4. This is a sign of true labor. The contractions increase in duration and intensity and begin in the back and radiate around to the abdomen. 5. It is a sign of true labor when the client is unable to walk during the contraction.

The nurse knows that a contraindication to the induction of labor is which of the following? 1. Placenta previa 2. Isoimmunization 3. Diabetes mellitus 4. Premature rupture of membranes

Answer: 1 Explanation: 1. Placenta previa is a contraindication to the induction of labor.

Which of the following is a major side effect of butorphanol tartrate (Stadol)? 1. Blurred vision 2. Agitation 3. Feelings of dysphoria 4. Drowsiness

Answer: 3 Explanation: 3. Feelings of dysphoria are a major side effect of Stardol.

Which client requires immediate intervention by the labor and delivery nurse? 1. Client at 8 cm, systolic blood pressure has increased 35 mm Hg 2. Client who delivered 1 hour ago with WBC of 50,000 3. Client at 5 cm with a respiratory rate of 22 between contractions 4. Client in active labor with polyuria

: 2 Explanation: 2. The white blood cell (WBC) count increases to 25,000/mm3 to 30,000/mm3 during labor and early postpartum. This count is abnormally high, and requires further assessment and provider notification.

A woman has been in labor for 16 hours. Her cervix is dilated to 3 cm and is 80% effaced. The fetal presenting part is not engaged. The nurse would suspect which of the following? 1. Breech malpresentation 2. Fetal demise 3. Cephalopelvic disproportion (CPD) 4. Abruptio placentae

Answer: 3 Explanation: 3. Cephalopelvic disproportion (CPD) prevents the presenting part from becoming engaged.

The home health nurse is admitting a client at 18 weeks who is pregnant with twins. Which nursing action is most important? 1. Teach the client about foods that are good sources of protein. 2. Assess the client's blood pressure in her upper right arm. 3. Determine whether the pregnancy is the result of infertility treatment. 4. Collect a cervicovaginal fetal fibronectin (fFN) specimen.

Answer: 1 Explanation: 1. A daily intake of 4000 kcal (minimum) and 135 g protein is recommended for a woman with normal-weight twins.

A woman is admitted to the birth setting in early labor. She is 3 cm dilated, -2 station, with intact membranes and FHR of 150 beats/min. Her membranes rupture spontaneously, and the FHR drops to 90 beats/min with variable decelerations. What would the initial response from the nurse be? 1. Perform a vaginal exam. 2. Notify the physician. 3. Place the client in a left lateral position. 4. Administer oxygen at 2 L per nasal cannula.

Answer: 1 Explanation: 1. A drop in fetal heart rate accompanied by variable decelerations is consistent with a prolapsed cord. The nurse would assess for prolapsed cord via vaginal examination.

The nurse auscultates the FHR and determines a rate of 112 beats/min. Which action is appropriate? 1. Inform the maternal client that the rate is normal. 2. Reassess the FHR in 5 minutes because the rate is low. 3. Report the FHR to the doctor immediately. 4. Turn the maternal client on her side and administer oxygen.

Answer: 1 Explanation: 1. A fetal heart rate of 112 beats/min. falls within the normal range of 110-160 beats/min., so there is no need to inform the doctor.

A cesarean section is ordered for a pregnant client. Because the client is to receive general anesthesia, what is the primary danger with which the nurse is concerned? 1. Fetal depression 2. Vomiting 3. Maternal depression 4. Uterine relaxation

Answer: 1 Explanation: 1. A primary danger of general anesthesia is fetal depression. The depression in the fetus is directly proportional to the depth and duration of the anesthesia.

The nurse is scheduling a client for an external cephalic version (ECV). Which finding in the client's chart requires immediate intervention? 1. Previous birth by cesarean 2. Frank breech ballotable 3. 37 weeks, complete breech 4. Failed ECV last week

Answer: 1 Explanation: 1. Any previous uterine scar is a contraindication to ECV. Prior scarring of the uterus may increase the risk of uterine tearing or uterine rupture.

A pregnant client is admitted to the emergency department with bleeding. The nurse realizes that the client might have placenta previa. Which signs would be suggestive of placenta previa? 1. Bright red vaginal bleeding 2. Sudden onset of vaginal bleeding 3. Firm and hard uterus 4. Change in the size of abdomen

Answer: 1 Explanation: 1. As the lower uterine segment contracts and dilates, the placental villi are torn from the uterine wall, causing bright red bleeding.

The nurse understands that the classic symptom of endometritis in a postpartum client is which of the following? 1. Purulent, foul-smelling lochia 2. Decreased blood pressure 3. Flank pain 4. Breast is hot and swollen

Answer: 1 Explanation: 1. Assessment findings consistent with endometritis are foul-smelling lochia, fever, uterine tenderness on palpation, lower abdominal pain, tachycardia, and chills.

The nurse has received end-of-shift reports in the high-risk maternity unit. Which client should the nurse see first? 1. The client at 26 weeks' gestation with placenta previa experiencing blood on toilet tissue after a bowel movement 2. The client at 30 weeks' gestation with placenta previa whose fetal monitor strip shows late decelerations 3. The client at 35 weeks' gestation with grade I abruptio placentae in labor who has a strong urge to push 4. The client at 37 weeks' gestation with pregnancy-induced hypertension whose membranes ruptured spontaneously

Answer: 1 Explanation: 1. Assessment of the woman with placenta previa must be ongoing to prevent or treat complications that are potentially lethal to the mother and fetus. Painless, bright red vaginal bleeding is the best diagnostic sign of placenta previa. This client is the highest priority.

The home health nurse is visiting a new mother whose baby was delivered by emergency cesarean after a car accident. The mother seems dazed, irritable, and unaware of her surroundings. She tells the nurse she has had trouble sleeping. What would the nurse suspect that the mother has? 1. Post-traumatic stress disorder 2. Postpartum blues 3. Postpartum psychosis 4. Disenfranchised grief

Answer: 1 Explanation: 1. Because of the traumatic nature of the birth and the client's symptoms, this condition is most likely post-traumatic stress disorder (PTSD). At particular risk for PTSD are women who have histories of prior trauma and/or prior psychiatric histories and women who undergo emergency cesarean sections.

The nurse knows that a baby born to a mother who had oligohydramnios could show signs of which of the following? 1. Respiratory difficulty 2. Hypertension 3. Heart murmur 4. Decreased temperature

Answer: 1 Explanation: 1. Because there is less fluid available for the fetus to use during fetal breathing movements, pulmonary hypoplasia may develop.

A nurse needs to evaluate the progress of a woman's labor. The nurse obtains the following data: cervical dilatation 6 cm; contractions mild in intensity, occurring every 5 minutes, with a duration of 30-40 seconds. Which clue in this data does not fit the pattern suggested by the rest of the clues? 1. Cervical dilatation 6 cm 2. Mild contraction intensity 3. Contraction frequency every 5 minutes 4. Contraction duration 30-40 seconds

Answer: 1 Explanation: 1. Cervical dilatation of 6 cm indicates the active phase of labor. During this phase the cervix dilates from about 4 to 7 cm and contractions and pain intensify.

The postpartum client who delivered 2 days ago has developed endometritis. Which entry would the nurse expect to find in this client's chart? 1. "Cesarean birth after extended labor with ruptured membranes." 2. "Unassisted childbirth and afterbirth." 3. "External fetal monitoring used throughout labor." 4. "The client has history of pregnancy-induced hypertension."

Answer: 1 Explanation: 1. Cesarean birth is the single most significant risk of postpartum endometritis as well as prolonged premature rupture of the amniotic membranes (PPROM).

A postpartum client has inflamed hemorrhoids. Which nursing intervention would be appropriate? 1. Encourage sitz baths. 2. Position the client in the supine position. 3. Avoid stool softeners. 4. Decrease fluid intake.

Answer: 1 Explanation: 1. Encouraging sitz baths is the correct approach because moist heat decreases inflammation and provides for comfort.

The client has been pushing for 3 hours, and the fetus is making a slow descent. The partner asks the nurse whether pushing for this long is normal. How should the nurse respond? 1. "Your baby is taking a little longer than average, but is making progress." 2. "First babies take a long time to be born. The next baby will be easier." 3. "The birth would go faster if you had taken prenatal classes and practiced." 4. "Every baby is different; there really are no norms for labor and birth."

Answer: 1 Explanation: 1. Establishing rapport and a trusting relationship and providing information that is true is best response.

A nurse is checking the postpartum orders. The doctor has prescribed bed rest for 6-12 hours. The nurse knows this is an appropriate order if the client had which type of anesthesia? 1. Spinal 2. Pudendal 3. General 4. Epidural

Answer: 1 Explanation: 1. Following the birth, the woman may be kept flat. Although the effectiveness of the supine position to avoid headache following a spinal is controversial, the physician's orders may include lying flat for 6 to 12 hours.

The nurse is caring for a laboring client with thrombocytopenia. During labor, it is determined that the client requires a cesarean delivery. The nurse is preparing the client for surgery, and should instruct the client that the recommended method of anesthesia is which of the following? 1. General anesthesia 2. Epidural anesthesia 3. Spinal anesthesia 4. Regional anesthesia

Answer: 1 Explanation: 1. General anesthesia will be recommended. Women with thrombocytopenia should avoid regional blocks.

An analgesic medication has been administered intramuscularly to a client in labor. How would the nurse evaluate if the medication was effective? 1. The client dozes between contractions. 2. The client is moaning during contractions. 3. The contractions decrease in intensity. 4. The contractions decrease in frequency.

Answer: 1 Explanation: 1. If the client dozes between contractions, the analgesic is effective. Analgesics decrease discomfort and increase relaxation.

The postpartum multipara is breastfeeding her new baby. The client states that she developed mastitis with her first child, and asks whether there is something she can do to prevent mastitis this time. What would the best response of the nurse be? 1. "Massage your breasts on a daily basis, and if you find a hardened area, massage it towards the nipple." 2. "Most first-time moms experience mastitis. It is really quite unusual for a woman having her second baby to get it again." 3. "Apply cabbage leaves to any areas that feel thickened or firm to relieve the swelling." 4. "Take your temperature once a day. This will help you to pick up the infection early, before it becomes severe."

Answer: 1 Explanation: 1. If the mother finds that one area of her breast feels distended or lumpy, she can massage the lumpy area toward the nipple as the infant nurses.

A client delivered 30 minutes ago. Which postpartal assessment finding would require close nursing attention? 1. A soaked perineal pad since the last 15-minute check 2. An edematous perineum 3. The client experiencing tremors 4. A fundus located at the umbilicus

Answer: 1 Explanation: 1. If the perineal pad becomes soaked in a 15-minute period or if blood pools under the buttocks, continuous observation is necessary. As long as the woman remains in bed during the first hour, bleeding should not exceed saturation of one pad.

The client gave birth to a 7 pound, 14 ounce female 30 minutes ago. The placenta has not yet delivered. Manual removal of the placenta is planned. What should the nurse prepare to do? 1. Start an IV of lactated Ringer's. 2. Apply anti-embolism stockings. 3. Bottle-feed the infant. 4. Send the placenta to pathology.

Answer: 1 Explanation: 1. In women who do not have an epidural in place, intravenous sedation may be required because of the discomfort caused by the procedure. An IV is necessary.

The nurse administered oxytocin 20 units at the time of placental delivery. Why was this primarily done? 1. To contract the uterus and minimize bleeding 2. To decrease breast milk production 3. To decrease maternal blood pressure 4. To increase maternal blood pressure

Answer: 1 Explanation: 1. Oxytocin is given to contract the uterus and minimize bleeding.

A woman has been having contractions since 4 a.m. At 8 a.m., her cervix is dilated to 5 cm. Contractions are frequent, and mild to moderate in intensity. Cephalopelvic disproportion (CPD) has been ruled out. After giving the mother some sedation so she can rest, what would the nurse anticipate preparing for? 1. Oxytocin induction of labor 2. Amnioinfusion 3. Increased intravenous infusion 4. Cesarean section

Answer: 1 Explanation: 1. Oxytocin is the drug of choice for labor augmentation or labor induction and may be administered as needed for hypotonic labor patterns.

The nurse is preparing to assess the fetus of a laboring client. Which assessment should the nurse perform first? 1. Perform Leopold maneuvers to determine fetal position. 2. Count the fetal heart rate between, during, and for 30 seconds following a uterine contraction (UC). 3. Dry the maternal abdomen before using the Doppler. 4. The diaphragm should be cooled before using the Doppler.

Answer: 1 Explanation: 1. Performing Leopold maneuvers is the first step.

The nurse is inducing the labor of a client with severe preeclampsia. As labor progresses, fetal intolerance of labor develops. The induction medication is turned off, and the client is prepared for cesarean birth. Which statement should the nurse include in her preoperative teaching? 1. "Because of your preeclampsia you are at higher risk for hypotension after an epidural anesthesia." 2. "Because of your preeclampsia you might develop hypertension after a spinal anesthesia." 3. "Because of your preeclampsia your baby might have decreased blood pressure after birth." 4. "Because of your preeclampsia

Answer: 1 Explanation: 1. Pregnancies complicated by preeclampsia are high-risk situations. The woman with mild preeclampsia usually may have the analgesia or anesthesia of choice, although the incidence of hypotension with epidural anesthesia is increased. If hypotension occurs with the epidural block, it provides further stress on an already compromised cardiovascular system.

A client is admitted to the birth setting in early labor. She is 3 cm dilated, -2 station, with intact membranes, and FHR of 150 bpm. Her membranes rupture spontaneously, and the FHR drops to 90 bpm with variable decelerations. What would the nurse's initial response be? 1. Perform a vaginal exam 2. Notify the physician 3. Place the client in a left lateral position 4. Administer oxygen at 2 L per nasal cannula

Answer: 1 Explanation: 1. Prolapsed umbilical cord can occur when the membranes rupture. The fetus is more likely to experience variable decelerations because the amniotic fluid is insufficient to keep pressure off the umbilical cord. A vaginal exam is the best way to confirm.

The nurse is reviewing charts of clients who underwent cesarean births by request in the last two years. The hospital is attempting to decrease costs of maternity care. What findings contribute to increased health care costs in clients undergoing cesarean birth by request? 1. Increased abnormal placenta implantation in subsequent pregnancies 2. Decreased use of general anesthesia with greater use of epidural anesthesia 3. Prolonged anemia, requiring blood transfusions every few months 4. Coordination of career projects of both partners leading to increased income

Answer: 1 Explanation: 1. Repeat cesarean births are associated with greater risks including increased incidence of abnormal placentation in subsequent pregnancies and the increased risk of mortality secondary to surgery, which would contribute to increased health care costs.

The community nurse is working with a client from Southeast Asia who has delivered her first child. Her mother has come to live with the family for several months. The nurse understands that the main role of the grandmother while visiting is to do which of the following? 1. Help the new mother by allowing her to focus on resting and caring for the baby. 2. Teach her son-in-law the right way to be a father because this is his first child. 3. Make sure that her daughter does not become abusive towards the infant. 4. Pass on the cultural values and beliefs to the newborn grandchild.

Answer: 1 Explanation: 1. Rest, seclusion, and dietary restraint practices in many traditional non-Western cultures (South Asian groups) are designed to assist the woman and her baby during postpartum vulnerable periods.

The nurse is teaching a class on vaginal birth after cesarean (VBAC). Which statement by a participant indicates that additional information is needed? 1. "Because the scar on my belly goes down from my navel, I am not a candidate for a VBAC." 2. "My first baby was in a breech position, so for this pregnancy, I can try a VBAC if the baby is head-down." 3. "Because my hospital is so small and in a rural area, they won't let me attempt a VBAC." 4. "The rate of complications from VBAC is lower than the rate of complications from a cesarean."

Answer: 1 Explanation: 1. Skin incision is not indicative of uterine incision. Only the uterine incision is a factor in deciding whether VBAC is advisable. The classic vertical incision was commonly done in the past and is associated with increased risk of uterine rupture in subsequent pregnancies and labor.

Two hours ago, a client at 39 weeks' gestation was 3 cm dilated, 40% effaced, and +1 station. Frequency of contractions was every 5 minutes with duration 40 seconds and intensity 50 mmHg. The current assessment is 4 cm dilated, 40% effaced, and +1 station. Frequency of contractions is now every 3 minutes with 40-50 seconds' duration and intensity of 40 mmHg. What would the priority intervention be? 1. Begin oxytocin after assessing for CPD. 2. Give terbutaline to stop the preterm labor. 3. Start oxygen at 8 L/min. 4. Have the anesthesiologist give the client an epidural.

Answer: 1 Explanation: 1. The client is having hypertonic contractions. Cephalopelvic disproportion (CPD) must be excluded. If CPD exists, oxytocin (Pitocin) augmentation should not be used. Oxytocin is the drug of choice for labor augmentation or labor induction.

The client delivered by cesarean birth 3 days ago and is being discharged. Which statement should the nurse include in the discharge teaching? 1. "If your incision becomes increasingly painful, call the doctor." 2. "It is normal for the incision to ooze greenish discharge in a few days." 3. "Increasing redness around the incision is a part of the healing process." 4. "A fever is to be expected because you had a surgical delivery."

Answer: 1 Explanation: 1. The client should call the doctor if the incision becomes increasingly painful. After cesarean delivery, wound infection is most often associated with concurrent endometritis. The wound is typically red, indurated, tender at the margins, and draining purulent exudate. Some women have cellulitis without actual purulent drainage.

The laboring client's fetal heart rate baseline is 120 beats per minute. Accelerations are present to 135 beats/min. During contractions, the fetal heart rate gradually slows to 110, and is at 120 by the end of the contraction. What nursing action is best? 1. Document the fetal heart rate. 2. Apply oxygen via mask at 10 liters. 3. Prepare for imminent delivery. 4. Assist the client into Fowler's position.

Answer: 1 Explanation: 1. The described fetal heart rate has a normal baseline; the presence of accelerations indicates adequate fetal oxygenation, and early decelerations are normal. No intervention is necessary.

To identify the duration of a contraction, the nurse would do which of the following? 1. Start timing from the beginning of one contraction to the completion of the same contraction. 2. Time between the beginning of one contraction and the beginning of the next contraction. 3. Palpate for the strength of the contraction at its peak. 4. Time from the beginning of the contraction to the peak of the same contraction.

Answer: 1 Explanation: 1. The duration of each contraction is measured from the beginning of the contraction to the completion of the contraction.

The fetal heart rate baseline is 140 beats/min. When contractions begin, the fetal heart rate drops suddenly to 120, and rapidly returns to 140 before the end of the contraction. Which nursing intervention is best? 1. Assist the client to change position. 2. Apply oxygen to the client at 2 liters per nasal cannula. 3. Notify the operating room of the need for a cesarean birth. 4. Determine the color of the leaking amniotic fluid.

Answer: 1 Explanation: 1. The fetus is exhibiting variable decelerations, which are caused by cord compression. Sometimes late or variable decelerations are due to the supine position of the laboring woman. In this case, the decrease in uterine blood flow to the fetus may be alleviated by raising the woman's upper trunk or turning her to the side to displace pressure of the gravid uterus on the inferior vena cava.

What is the major adverse side effect of epidural anesthesia? 1. Maternal hypotension 2. Decrease in variability of the FHR 3. Vertigo 4. Decreased or absent respiratory movements

Answer: 1 Explanation: 1. The major adverse effect of epidural anesthesia is maternal hypotension caused by a spinal blockade, which lowers peripheral resistance, decreases venous return to the heart, and subsequently lessens cardiac output and lowers blood pressure.

After administration of an epidural anesthetic to a client in active labor, it is most important to assess the mother immediately for which of the following? 1. Hypotension 2. Headache 3. Urinary retention 4. Bradycardia

Answer: 1 Explanation: 1. The most common complication of an epidural is maternal hypotension.

What is one of the most common initial signs of nonreassuring fetal status? 1. Meconium-stained amniotic fluid 2. Cyanosis 3. Dehydration 4. Arrest of descent

Answer: 1 Explanation: 1. The most common initial signs of nonreassuring fetal status are meconium-stained amniotic fluid and changes in the fetal heart rate (FHR).

What is the most significant maternal risk factor for preterm birth? 1. Previous preterm birth 2. Smoking 3. Stress 4. Substance abuse

Answer: 1 Explanation: 1. The most significant maternal risk factor for preterm birth is a previous preterm birth.

Which of the following behaviors noted in the postpartum client would require the nurse to assess further? 1. Responds hesitantly to infant cries. 2. Expresses satisfaction about the sex of the baby. 3. Friends and family visit the client and give advice. 4. Talks to and cuddles with the infant frequently.

Answer: 1 Explanation: 1. The mother tends to respond verbally to any sounds emitted by the newborn, such as cries, coughs, sneezes, and grunts. Responding hesitantly to infant cries might need further assessment to determine what the mother is feeling.

After being in labor for several hours with no progress, a client is diagnosed with CPD (cephalopelvic disproportion), and must have a cesarean section. The client is worried that she will not be able to have any future children vaginally. After sharing this information with her care provider, the nurse would anticipate that the client would receive what type of incision? 1. Transverse 2. Infraumbilical midline 3. Classic 4. Vertical

Answer: 1 Explanation: 1. The transverse incision is made across the lowest and narrowest part of the abdomen and is the most common lower uterine segment incision.

The laboring client is complaining of tingling and numbness in her fingers and toes, dizziness, and spots before her eyes. The nurse recognizes that these are clinical manifestations of which of the following? 1. Hyperventilation 2. Seizure auras 3. Imminent birth 4. Anxiety

Answer: 1 Explanation: 1. These symptoms all are consistent with hyperventilation.

The client delivered vaginally 2 hours ago after receiving an epidural analgesia. She has a slight tingling sensation in both lower extremities, but normal movement. She sustained a second-degree perineal laceration. Her perineum is edematous and ecchymotic. What should the nurse include in the plan of care for this client? 1. Assist the client to the bathroom in 2 hours to void. 2. Place a Foley catheter now. 3. Apply warm packs to the perineum three times a day. 4. Allow the client to rest for the next 8 hours.

Answer: 1 Explanation: 1. This client is at risk for urinary retention and bladder overdistention. Overdistention occurs postpartum when the woman is unable to empty her bladder, usually because of trauma or the effects of anesthesia. After the effects of anesthesia have worn off, if the woman cannot void, postpartum urinary retention is highly indicative of a urinary tract infection (UTI). Assisting the client to the bathroom is the most likely intervention that will prevent urinary retention.

The nurse is admitting a client for a cerclage procedure. The client asks for information about the procedure. What is the nurse's most accurate response? 1. "A stitch is placed in the cervix to prevent a spontaneous abortion or premature birth." 2. "The procedure is done during the third trimester." 3. "Cerclage is always placed after the cervix has dilated and effaced." 4. "An uncomplicated elective cerclage may is done on inpatient basis."

Answer: 1 Explanation: 1. This is the correct description of cerclage.

The labor nurse is caring for a client at 38 weeks' gestation who has been diagnosed with symptomatic placenta previa. Which physician order should the nurse question? 1. Begin oxytocin drip rate at 0.5 milliunits/min. 2. Assess fetal heart rate every 10 minutes. 3. Weigh all vaginal pads. 4. Assess hematocrit and hemoglobin.

Answer: 1 Explanation: 1. This order should be questioned, as this client is not a good candidate for labor induction.

By inquiring about the expectations and plans that a laboring woman and her partner have for the labor and birth, the nurse is primarily doing which of the following? 1. Recognizing the client as an active participant in her own care. 2. Attempting to correct any misinformation the client might have received. 3. Acting as an advocate for the client. 4. Establishing rapport with the client.

Answer: 1 Explanation: 1. Understanding the couple's expectations and plans helps the nurse provide optimal nursing care and facilitate the best possible birth experience.

The client has experienced a postpartum hemorrhage at 6 hours postpartum. After controlling the hemorrhage, the client's partner asks what would cause a hemorrhage. How should the nurse respond? 1. "Sometimes the uterus relaxes and excessive bleeding occurs." 2. "The blood collected in the vagina and poured out when your partner stood up." 3. "Bottle-feeding prevents the uterus from getting enough stimulation to contract." 4. "The placenta had embedded in the uterine tissue abnormally."

Answer: 1 Explanation: 1. Uterine atony (relaxation of the uterus) is the leading cause of early postpartum hemorrhage, accounting for over 50% of postpartum hemorrhage cases.

The client tells the nurse that she has come to the hospital so that her baby's position can be changed. The nurse would begin to organize the supplies needed to perform which procedure? 1. A version 2. An amniotomy 3. Leopold maneuvers 4. A ballottement

Answer: 1 Explanation: 1. Version, or turning the fetus, is a procedure used to change the fetal presentation by abdominal or intrauterine manipulation.

The laboring client presses the call light and reports that her water has just broken. What would the nurse's first action be? 1. Check fetal heart tones. 2. Encourage the mother to go for a walk. 3. Change bed linens. 4. Call the physician.

Answer: 1 Explanation: 1. When the membranes rupture, the nurse notes the color and odor of the amniotic fluid and the time of rupture and immediately auscultates the FHR.

Two hours after delivery, a client's fundus is boggy and has risen to above the umbilicus. What is the first action the nurse would take? 1. Massage the fundus until firm 2. Express retained clots 3. Increase the intravenous solution 4. Call the physician

Answer: 1 Explanation: 1. When the uterus becomes boggy, pooling of blood occurs within it, resulting in the formation of clots. Anything left in the uterus prevents it from contracting effectively. Thus if it becomes boggy or appears to rise in the abdomen, the fundus should be massaged until firm.

The client has asked the nurse why her cervix has only changed from 1 to 2 cm in 3 hours of contractions occurring every 5 minutes. What is the nurse's best response to the client? 1. "Your cervix has also effaced, or thinned out, and that change in the cervix is also labor progress." 2. "When your perineal body thins out, your cervix will begin to dilate much faster than it is now." 3. "What did you expect? You've only had contractions for a few hours. Labor takes time." 4. "The hormones that cause labor to begin are just getting to be at levels that will change your cervix."

Answer: 1 Explanation: 1. With each contraction, the muscles of the upper uterine segment shorten and exert a longitudinal traction on the cervix, causing effacement. Effacement is the taking up (or drawing up) of the internal os and the cervical canal into the uterine side walls.

A 26-year-old client is having her initial prenatal appointment. The client reports to the nurse that she suffered a pelvic fracture in a car accident 3 years ago. The client asks whether her pelvic fracture might affect her ability to have a vaginal delivery. What response by the nurse is best? 1. "It depends on how your pelvis healed." 2. "You will need to have a cesarean birth." 3. "Please talk to your doctor about that." 4. "You will be able to delivery vaginally."

Answer: 1 Explanation: 1. Women with a history of pelvic fractures may also be at risk for cephalopelvic disproportion (CPD).

A client attending a prenatal class asks why episiotomies are performed. The nurse explains that risk factors that predispose women to episiotomies include which of the following? Note: Credit will be given only if all correct choices and no incorrect choices are selected. Select all that apply. 1. Large or macrosomic fetus 2. Use of forceps 3. Shoulder dystocia 4. Maternal health 5. Shorter second stage

Answer: 1, 2, 3 Explanation: 1. A large fetus places a woman at risk for an episiotomy to prevent lacerations. 2. Use of forceps or vacuum extractor is a risk factor that predisposes women to episiotomies. 3. Shoulder dystocia is a risk factor that predisposes women to episiotomies.

A cesarean section is ordered for the laboring client with whom the nurse has worked all shift. The client will receive general anesthesia. The nurse knows that potential complications of general anesthesia include which of the following? Note: Credit will be given only if all correct choices and no incorrect choices are selected. Select all that apply. 1. Fetal depression that is directly proportional to the depth and duration of the anesthesia 2. Poor fetal metabolism of anesthesia, which inhibits use with preterm infants 3. Uterine relaxation 4. Increased gastric motility 5. Itching of the face and neck

Answer: 1, 2, 3 Explanation: 1. A primary danger of general anesthesia is fetal depression, because the medication reaches the fetus in about 2 minutes. The depression is directly proportional to the depth and duration of anesthesia. 2. The poor fetal metabolism of general anesthetic agents is similar to that of analgesic agents administered during labor. General anesthesia is not advocated when the fetus is considered to be at high risk, particularly in preterm birth. 3. Most general anesthetic agents cause some degree of uterine relaxation.

The labor nurse would not encourage a mother to bear down until the cervix is completely dilated, to prevent which of the following? Note: Credit will be given only for all correct choices and no incorrect choices. Select all that apply. 1. Maternal exhaustion 2. Cervical edema 3. Tearing and bruising of the cervix 4. Enhanced perineal thinning 5. Having to perform an episiotomy

Answer: 1, 2, 3 Explanation: 1. If the cervix is not completely dilated, maternal exhaustion can occur. 2. If the cervix is not completely dilated, cervical edema can occur. 3. If the cervix is not completely dilated, tearing and bruising of the cervix can occur.

The nurse is caring for a client who could be at risk for uterine rupture. The nurse is monitoring the fetus closely for which of the following? Note: Credit will be given only if all correct choices and no incorrect choices are selected. Select all that apply. 1. Late decelerations 2. Bradycardia 3. Loss of ability to determine fetal station 4. Tachycardia 5. Early decelerations

Answer: 1, 2, 3 Explanation: 1. Late decelerations could be seen with uterine rupture. 2. Bradycardia is seen if there is uterine rupture. 3. The uterus is not holding the fetus in place anymore if the uterus ruptures.

A nurse suspects that a postpartum client has mastitis. Which data support this assessment? Note: Credit will be given only if all correct choices and no incorrect choices are selected. Select all that apply. 1. Shooting pain between breastfeedings 2. Late onset of nipple pain 3. Pink, flaking, pruritic skin of the affected nipple 4. Nipple soreness when the infant latches on 5. Pain radiating to the underarm area from the breast

Answer: 1, 2, 3 Explanation: 1. Mastitis is characterized by shooting pain between feedings, often radiating to the chest wall. 2. Mastitis is characterized by late-onset nipple pain. 3. The skin of the affected breast becomes pink, flaking, and pruritic. Page Ref: 983

A client is admitted to the labor and delivery unit in active labor. What nursing diagnoses might apply to the client with suspected abruptio placentae? Note: Credit will be given only if all correct choices and no incorrect choices are selected. Select all that apply. 1. Fluid Volume, Deficient, Risk for, related to hypovolemia secondary to excessive blood loss 2. Tissue Perfusion: Peripheral, Ineffective, related to blood loss secondary to uterine atony following birth 3. Anxiety related to concern for own personal status and the baby's safety 4. Knowledge, Deficient related to lack of information about inherited genetic defects 5. Alteration in Respiratory Function related to blood loss

Answer: 1, 2, 3 Explanation: 1. Maternal and perinatal fetal mortality are concerns due to hypoxia. 2. Maternal and perinatal fetal mortality are concerns due to blood loss. 3. This mother would be anxious for herself and her baby.

A postpartal client recovering from deep vein thrombosis is being discharged. What areas of teaching on self-care and anticipatory guidance should the nurse discuss with the client? Note: Credit will be given only if all correct choices and no incorrect choices are selected. Select all that apply. 1. Avoid crossing the legs. 2. Avoid prolonged standing or sitting. 3. Take frequent walks. 4. Take a daily aspirin dose of 650 mg. 5. Avoid long car trips.

Answer: 1, 2, 3 Explanation: 1. Women should be taught to avoid prolonged standing or sitting in one position or sitting with legs crossed. 2. Women should be taught to avoid prolonged standing or sitting in one position or sitting with legs crossed. 3. Women should be advised to avoid a sedentary lifestyle and to exercise as much as possible (walking is ideal).

The nurse caring for a client in labor anticipates fetal macrosomia and shoulder dystocia. Appropriate management of shoulder dystocia is essential in order to prevent which fetal complications? Note: Credit will be given only if all correct choices and no incorrect choices are selected. Select all that apply. 1. Brachial plexus injury 2. Fractured clavicle 3. Asphyxia 4. Neurological damage 5. Puerperal infection

Answer: 1, 2, 3, 4 Explanation: 1. Brachial plexus injury occurs due to improper or excessive traction applied to the fetal head. 2. Complications in macrosomia include fractured clavicles. 3. Complications in macrosomia include asphyxia of the fetus. 4. Neurological damage is a complication of macrosomia.

What fetal factors require a cesarean birth? 1. Severe intrauterine growth restriction (IUGR) 2. Fetal anomalies 3. Unfavorable fetal position or presentation 4. Preterm birth 5. Lack of maternal attachment

Answer: 1, 2, 3, 4 Explanation: 1. Fetal factors such as severe intrauterine growth restriction (IUGR), preterm birth, fetal anomalies, nonreassuring fetal status, or unfavorable fetal position or presentation require cesarean birth. 2. Fetal factors such as severe intrauterine growth restriction (IUGR), preterm birth, fetal anomalies, nonreassuring fetal status, or unfavorable fetal position or presentation require cesarean birth. 3. Fetal factors such as severe intrauterine growth restriction (IUGR), preterm birth, fetal anomalies, nonreassuring fetal status, or unfavorable fetal position or presentation require cesarean birth. 4. Fetal factors such as severe intrauterine growth restriction (IUGR), preterm birth, fetal anomalies, nonreassuring fetal status, or unfavorable fetal position or presentation require cesarean birth.

The nurse is presenting a class on preterm labor, its causes, and treatments to a group of newly pregnant couples. Which statements regarding preterm labor are true? Note: Credit will be given only of all correct choices and no incorrect choices are selected. Select all that apply. 1. Antepartum hemorrhage can cause preterm labor. 2. Trauma can cause preterm labor. 3. Infection can cause preterm labor. 4. Magnesium sulfate is a drug used to stop contractions. 5. Sedatives and narcotics may be given to stop labor.

Answer: 1, 2, 3, 4 Explanation: 1. Hemorrhage from placenta previa or abruption can cause preterm labor. 2. Trauma to the abdomen or uterus can cause preterm labor. 3. Infections such as urinary tract infections can cause preterm labor. 4. Magnesium sulfate acts as a CNS depressant by decreasing the quantity of acetylcholine released by motor nerve impulses and thereby blocking neuromuscular transmission.

The nurse is caring for a client who is having fetal tachycardia. The nurse knows that possible causes include which of the following? Note: Credit will be given only if all correct choices and no incorrect choices are selected. Select all that apply. 1. Maternal dehydration 2. Maternal hyperthyroidism 3. Fetal hypoxia 4. Prematurity 5. Anesthesia or regional analgesia

Answer: 1, 2, 3, 4 Explanation: 1. Maternal dehydration can cause fetal tachycardia. 2. Maternal hyperthyroidism can cause fetal tachycardia. 3. Fetal tachycardia can indicate fetal hypoxia. 4. Prematurity can cause fetal tachycardia.

During a visit to the obstetrician, a pregnant client questions the nurse about the potential need for an amniotomy. The nurse explains that an amniotomy is performed to do which of the following? Note: Credit will be given only if all correct choices and no incorrect choices are selected. Select all that apply. 1. Stimulate the beginning of labor 2. Augment labor progression 3. Allow application of an internal fetal electrode 4. Allow application of an external fetal monitor 5. Allow insertion of an intrauterine pressure catheter

Answer: 1, 2, 3, 5 Explanation: 1. Amniotomy is the artificial rupture of the amniotic membranes and can be used to induce labor. 2. Amniotomy can be done to augment labor. 3. Amniotomy allows access to the fetus in order to apply an internal fetal electrode to the fetal scalp. 5. Amniotomy may be performed during labor to allow an intrauterine pressure catheter to be inserted.

Childbirth preparation offers several advantages including which of the following? Note: Credit will be given only for all correct choices and no incorrect choices. Select all that apply. 1. It helps a pregnant woman and her support person understand the choices in the birth setting. 2. It promotes awareness of available options. 3. It provides tools for a pregnant woman and her support person to use during labor and birth. 4. Women who receive continuous support during labor require more analgesia, and have more cesarean and instrument births. 5. Each method has been shown to shorten labor.

Answer: 1, 2, 3, 5 Explanation: 1. Childbirth preparation offers several advantages. It helps a pregnant woman and her support person understand the choices in the birth setting, promotes awareness of available options, and provides tools for them to use during labor and birth. 2. Childbirth preparation offers several advantages. It helps a pregnant woman and her support person understand the choices in the birth setting, promotes awareness of available options, and provides tools for them to use during labor and birth. 3. Childbirth preparation offers several advantages. It helps a pregnant woman and her support person understand the choices in the birth setting, promotes awareness of available options, and provides tools for them to use during labor and birth. 5. Childbirth preparation offers several advantages. Each method has been shown to shorten labor.

The nurse is teaching a prenatal class about postpartum changes. The nurse explains that factors that might interfere with uterine involution include which of the following? Note: Credit will be given only if all correct choices and no incorrect choices are selected. Select all that apply. 1. Prolonged labor 2. Difficult birth 3. Full bladder 4. Breastfeeding 5. Infection

Answer: 1, 2, 3, 5 Explanation: 1. During prolonged labor, the muscles relax because of prolonged time of contraction during labor. 2. During a difficult birth, the uterus is manipulated excessively, causing fatigue. 3. As the uterus is pushed up and usually to the right, pressure on a full bladder interferes with effective uterine contraction. 5. Inflammation and infection interfere with uterine muscle's ability to contract effectively.

Upon assessing the FHR tracing, the nurse determines that there is fetal tachycardia. The fetal tachycardia would be caused by which of the following? Note: Credit will be given only if all correct choices and no incorrect choices are selected. Select all that apply. 1. Early fetalhypoxia 2. Prolonged fetal stimulation 3. Fetal anemia 4. Fetal sleep cycle 5. Infection

Answer: 1, 2, 3, 5 Explanation: 1. Early fetal hypoxia can cause fetal tachycardia. 2. Prolonged fetal stimulation can cause fetal tachycardia. 3. Fetal anemia can cause fetal tachycardia. 5. Infection can cause fetal tachycardia.

For what common side effects of epidural anesthesia should the nurse watch? Note: Credit will be given only if all correct choices and no incorrect choices are selected. Select all that apply. 1. Elevated maternal temperature 2. Urinary retention 3. Nausea 4. Long-term back pain 5. Local itching

Answer: 1, 2, 3, 5 Explanation: 1. Elevated maternal temperature is a potential side effect of epidural anesthesia. 2. Urinary retention is a potential side effect of epidural anesthesia. 3. Nausea is a potential side effect of epidural anesthesia. 5. Pruritus may occur at any time during the epidural infusion. It usually appears first on the face, neck, or torso and is generally the result of the agent used in the epidural infusion. Benadryl, an antihistamine, can be administered to manage pruritus.

) In caring for a client with a uterine rupture, the nurse determines which nursing diagnoses to be appropriate? Note: Credit will be given only if all correct choices and no incorrect choices are selected. Select all that apply. 1. Gas Exchange, Impaired 2. Fear related to unknown outcome 3. Coping, Ineffective 4. Mobility: Physical, Impaired 5. Anxiety

Answer: 1, 2, 3, 5 Explanation: 1. Gas Exchange, Impaired diagnosis could apply to both mother and fetus. 2. The client would experience fear related to an unknown outcome. 3. Ineffective coping would be due to emergent situation secondary to uterine rupture. 5. There will be anxiety related to emergency procedures and unknown fetal outcome.

Which interventions can the nurse utilize to provide continuity of care for the postpartal client who experienced a complication and is now ready to return home? Note: Credit will be given only if all correct choices and no incorrect choices are selected. Select all that apply. 1. Encourage the client to take advantage of home visits. 2. Make telephone calls as a follow-up to check on the client and newborn. 3. Provide information about postpartal support groups. 4. Refer to mental health professionals to help screen the client for any mental health problems as a result of the complications experienced in the hospital. 5. Supply information about postpartum expectations designed to meet the specific needs of a variety of families.

Answer: 1, 2, 3, 5 Explanation: 1. Home visits, especially for early discharge families, are invaluable in fostering positive adjustments for the new family. 2. Telephone follow-up at 2 to 3 weeks postpartum to ask whether the mother is experiencing difficulties is also helpful. 3. Support groups in which child care is available can be an invaluable community service for the postpartum client. 5. Social support teaching guides are available to assist in helping postpartum women explore their needs for postpartum support.

The nurse is presenting a class for nursing students on multiple-gestation pregnancy. Which statements about multiple-gestation pregnancies are accurate? Note: Credit will be given only if all correct choices and no incorrect choices are selected. Select all that apply. 1. Hypertension is a major maternal complication. 2. Gestational diabetes occurs more often. 3. Maternal anemia occurs. 4. Pulmonary embolism is 12 times more likely to develop during pregnancy with multiple gestations. 5. Multiple gestations are more likely to acquire HELLP.

Answer: 1, 2, 3, 5 Explanation: 1. Hypertension is a complication in multiple-gestation pregnancies. 2. Gestational diabetes occurs more often in multiple gestations. 3. Maternal anemia occurs because of demands of the multiple gestations. 5. Multiple gestations are more likely to acquire HELLP (hemolytic anemia, elevated liver enzymes, and low platelet count) syndrome, a complication resulting from eclampsia or preeclampsia.

The nurse is teaching a class on reading a fetal monitor to nursing students. The nurse explains that bradycardia is a fetal heart rate baseline below 110 and can be caused by which of the following? Note: Credit will be given only if all correct choices and no incorrect choices are selected. Select all that apply. 1. Maternal hypotension 2. Prolonged umbilical cord compression 3. Fetal dysrhythmia 4. Central nervous system malformation 5. Late fetal asphyxia

Answer: 1, 2, 3, 5 Explanation: 1. Maternal hypotension results in decreased blood flow to the fetus. 2. Cord compression can cause fetal bradycardia. 3. This will cause fetal bradycardia if there is a fetal heart block. 5. This is a depression of myocardial activity.

At her 6-week postpartum checkup, a new mother voices concerns to the nurse. She states that she is finding it hard to have time alone to even talk on the phone without interruption. Her family lives in another state, and she has contact with them only by phone. She is still having difficulty getting enough sleep and worries that she will not be a good mother. Appropriate nursing interventions would include providing which of the following? Note: Credit will be given only if all correct choices and no incorrect choices are selected. Select all that apply. 1. Anticipatory guidance about the realities of being a parent. 2. Parenting literature and reference manuals. 3. Phone numbers and locations of local parenting groups. 4. Referral for specialized interventions related to postpartum blues. 5. Phone numbers and names of postpartum doulas.

Answer: 1, 2, 3, 5 Explanation: 1. Postpartum nurses need to be aware of the long-term adjustments and stresses that the childbearing family faces as its members adjust to new and different roles. 2. Postpartum nurses need to be aware of the long-term adjustments and stresses that the childbearing family faces as its members adjust to new and different roles. 3. New mother support groups are helpful for women who lack a social support system. 5. Postpartum doulas are professionals trained to help the new mother after the birth of the baby.

) The nurse has completed the physical assessment of a client in early labor, and proceeds with the social assessment. A social history of the client would include which of the following? Note: Credit will be given only if all correct choices and no incorrect choices are selected. Select all that apply. 1. Use of drugs and alcohol 2. Family violence or sexual assault 3. Current living situation 4. Type of insurance 5. Availability of resources

Answer: 1, 2, 3, 5 Explanation: 1. Risk factors such as the use of drugs or alcohol during the pregnancy can influence the labor and birth. 2. It is imperative to ask the woman about domestic violence and to assess any degree of psychologic or physical harm, either potential or real. 3. A social assessment includes asking about the woman's current living situation. This dialog provides an opportunity for the nurse to continue to build support, to provide information when requested, and to be direct yet supportive. 5. A social assessment includes asking about resources available to the family.

A prenatal client asks the nurse about conditions that would necessitate a cesarean delivery. The nurse explains that cesarean delivery generally is performed in the presence of which of the following? Note: Credit will be given only if all correct choices and no incorrect choices are selected. Select all that apply. 1. Complete placenta previa 2. Placental abruption 3. Umbilical cord prolapse 4. Precipitous labor 5. Failure to progress

Answer: 1, 2, 3, 5 Explanation: 1. When the placenta completely covers the uterine opening, a cesarean is performed. 2. Premature separation of the placenta from the uterine wall requires an immediate cesarean. 3. A prolapsed cord is an emergency requiring an immediate cesarean. 5. Failure to progress in labor can necessitate a cesarean birth.

Premonitory signs of labor include which of the following? Note: Credit will be given only for all correct choices and no incorrect choices. Select all that apply. 1. Braxton Hicks contractions 2. Cervical softening and effacement 3. Weight gain 4. Rupture of membranes 5. Sudden loss of energy

Answer: 1, 2, 4 Explanation: 1. A premonitory sign of labor includes Braxton Hicks contractions. 2. A premonitory sign of labor includes cervical softening and effacement. 4. A premonitory sign of labor includes rupture of membranes.

The nurse is caring for a client in labor. Which signs and symptoms would indicate the client is progressing into the second stage of labor? Note: Credit will be given only for all correct choices and no incorrect choices. Select all that apply. 1. Bulging perineum 2. Increased bloody show 3. Spontaneous rupture of the membranes 4. Uncontrollable urge to push 5. Inability to breathe through contractions

Answer: 1, 2, 4 Explanation: 1. As the fetal head continues its descent, the perineum begins to bulge, flatten, and move anteriorly. 2. Bloody show increases as a woman enters the second stage of labor. 4. As the fetal head descends, the woman has the urge to push because of pressure of the fetal head on the sacral and obturator nerves.

Risk factors for tachysystole include which of the following? Note: Credit will be given only if all correct choices and no incorrect choices are selected. Select all that apply. 1. Cocaine use 2. Placental abruption 3. Low-dose oxytocin titration regimens 4. Uterine rupture 5. Smoking

Answer: 1, 2, 4 Explanation: 1. Cocaine use is a risk factor for tachysystole. 2. Placental abruption is a risk factor for tachysystole. 4. Uterine rupture is a risk factor for tachysystole.

The client is undergoing an emergency cesarean birth for fetal bradycardia. The client's partner has not been allowed into the operating room. What can the nurse do to alleviate the partner's emotional distress? Note: Credit will be given only if all correct choices and no incorrect choices are selected. Select all that apply. 1. Allow the partner to wheel the baby's crib to the newborn nursery. 2. Allow the partner to be near the operating room where the newborn's first cry can be heard. 3. Have the partner wait in the client's postpartum room. 4. Encourage the partner to be in the nursery for the initial assessment. 5. Teach the partner how to take the client's blood pressure.

Answer: 1, 2, 4 Explanation: 1. Effective measures include allowing the partner to take the baby to the nursery. 2. Effective measures include allowing the partner to be in a place near the operating room, where the newborn's first cry can be heard. 4. Effective measures include involving the partner in postpartum care in the recovery room.

A full-term infant has just been born. Which interventions should the nurse perform first? Note: Credit will be given only if all correct choices and no incorrect choices are selected. Select all that apply. 1. Placing the infant in a radiant-heated unit 2. Suctioning the infant with a bulb syringe 3. Wrapping the infant in a blanket 4. Evaluating the newborn using the Apgar system 5. Offering a feeding of 5% glucose water

Answer: 1, 2, 4 Explanation: 1. If the newborn is placed in a radiant-heated unit, he or she is dried, laid on a dry blanket, and left uncovered under the radiant heat. 2. Newborns are suctioned with a bulb syringe to clear mucus from the newborn's mouth. 4. The purpose of the Apgar score is to evaluate the physical condition of the newborn at birth.

Which strategies would the nurse utilize to promote culturally competent care for the postpartum client? Note: Credit will be given only if all correct choices and no incorrect choices are selected. Select all that apply. 1. Examine one's own cultural beliefs, biases, stereotypes, and prejudices. 2. Respect the values and beliefs of others. 3. Limit the alternative food choices offered clients to minimize conflicts. 4. Incorporate the family's cultural practices into the care. 5. Evaluate whether the family's cultural practices fit into Western norms.

Answer: 1, 2, 4 Explanation: 1. It is important for nurses to recognize that they are approaching their patient's care from their own perspective and that, to individualize care for each mother, they need to assess the woman's preferences, her level of acculturation and assimilation to Western culture, her linguistic abilities, and her educational level. 2. It is important for nurses to recognize that they are approaching their patient's care from their own perspective and that, to individualize care for each mother, they need to assess the woman's preferences, her level of acculturation and assimilation to Western culture, her linguistic abilities, and her educational level. 4. The nurse should have the mother exercise her choices when possible and support those choices, with the help of cultural awareness and a sound knowledge base.

When caring for a laboring client with oligohydramnios, what should the nurse be aware of? Note: Credit will be given only if all correct choices and no incorrect choices are selected. Select all that apply. 1. Increased risk of cord compression 2. Decreased variability 3. Labor progress is often more rapid than average 4. Presence of periodic decelerations 5. During gestation, fetal skin and skeletal abnormalities can occur

Answer: 1, 2, 4, 5 Explanation: 1. During the labor and birth, the lessened amounts of fluid reduce the cushioning effect for the umbilical cord, and cord compression is more likely to occur. 2. The nurse should evaluate the EFM tracing for the presence of nonperiodic decelerations or other nonreassuring signs (such as increasing or decreasing baseline, decreased variability, or presence of periodic decelerations). 4. The nurse should evaluate the EFM tracing for the presence of nonperiodic decelerations or other nonreassuring signs (such as increasing or decreasing baseline, decreased variability, or presence of periodic decelerations). 5. During the gestational period, fetal skin and skeletal abnormalities may occur because fetal movement is impaired as a result of inadequate amniotic fluid volume.

The nurse is caring for a client who is showing a sinusoidal fetal heart rate pattern on the monitor. The nurse knows that possible causes for this pattern include which of the following? Note: Credit will be given only if all correct choices and no incorrect choices are selected. Select all that apply. 1. Fetal anemia 2. Chronic fetal bleeding 3. Maternal hypotension 4. Twin-to-twin transfusion 5. Umbilical cord occlusion

Answer: 1, 2, 4, 5 Explanation: 1. Fetal anemia can cause a sinusoidal heart rate. 2. Chronic fetal bleeding can cause a sinusoidal heart rate. 4. Twin-to-twin transfusion will cause a sinusoidal heart rate. 5. Umbilical cord occlusion can cause a sinusoidal fetal heart rate.

The nurse is assessing a client before administering an analgesic. What are some of the factors the nurse should consider? Note: Credit will be given only if all correct choices and no incorrect choices are selected. Select all that apply. 1. The client is willing to receive medication after being advised about it. 2. The client's vital signs are stable. 3. The partner agrees to use of the medication. 4. The client has no known allergies to the medication. 5. The client is aware of the contraindications of the medication.

Answer: 1, 2, 4, 5 Explanation: 1. Medication should be explained to the client before it is administered. 2. Vital signs need to be stable before any analgesic medication is administered. 4. Ask the client about allergies before administering any medications. 5. Clients should always be aware of the contraindications of the medication.

When preparing for and performing an assessment of the postpartum client, which of the following would the nurse do? Note: Credit will be given only if all correct choices and no incorrect choices are selected. Select all that apply. 1. Ask the client to void before assessing the uterus. 2. Inform the client of the need for regular assessments. 3. Defer client teaching to another time. 4. Perform the procedures as gently as possible. 5. Take precautions to prevent exposure to body fluids.

Answer: 1, 2, 4, 5 Explanation: 1. Palpating the fundus when the woman has a full bladder may give false information about the progress of involution. Ask the woman to void before assessment. 2. The nurse should provide an explanation of the purposes of regular assessment to the woman. 4. The woman should be relaxed before starting, and procedures should be performed as gently as possible, to avoid unnecessary discomfort. 5. Gloves should be worn before starting the assessment.

Which findings would indicate the presence of a perineal wound infection? Note: Credit will be given only if all correct choices and no incorrect choices are selected. Select all that apply. 1. Redness 2. Tender at the margins 3. Vaginal bleeding 4. Hardened tissue 5. Purulent drainage

Answer: 1, 2, 4, 5 Explanation: 1. Redness is a classic sign of a perineal wound infection. 2. The wound is typically red, indurated, tender at the margins, and draining purulent exudate. 4. The wound is typically red, indurated, tender at the margins, and draining purulent exudate. 5. Purulent drainage is a classic signs of a perineal wound infection.

When caring for a 13-year-old client in labor, how would the nurse provide sensitive care? Note: Credit will be given only if all correct choices and no incorrect choices are selected. Select all that apply. 1. Using simple and concrete instructions 2. Providing soothing encouragement and comfort measures 3. Making all decisions for the client when she expresses a feeling of helplessness 4. Deciding whom the client should allow in the room 5. Providing encouragement and support of the client's decisions

Answer: 1, 2, 5 Explanation: 1. A client at this developmental stage will need concrete and simplified instructions. 2. Touch, soothing encouragement, and measures to promote her comfort help her maintain control and meet her needs for dependence. 5. Establishing rapport without recrimination will provide emotional support and encouragement.

Risk factors associated with increased risk of thromboembolic disease include which of the following? Note: Credit will be given only if all correct choices and no incorrect choices are selected. Select all that apply. 1. Diabetes mellitus 2. Varicose veins 3. Hypertension 4. Adolescent pregnancy 5. Malignancy

Answer: 1, 2, 5 Explanation: 1. Diabetes mellitus is a risk factor for thromboembolic disease. 2. Varicose veins are a risk factor for thromboembolic disease. 5. Malignancy is a risk factor for thromboembolic disease.

The client presents to the labor and delivery unit stating that her water broke 2 hours ago. Indicators of normal labor include which of the following? Note: Credit will be given only if all correct choices and no incorrect choices are selected. Select all that apply. 1. Fetal heart rate of 130 with average variability 2. Blood pressure of 130/80 3. Maternal pulse of 160 4. Protein of +1 in urine 5. Odorless, clear fluid on underwear

Answer: 1, 2, 5 Explanation: 1. Fetal heart rate (FHR) of 110-160 with average variability is a normal indication. 2. Maternal vital sign of blood pressure below 140/90 is a normal indication. 5. Fluid clear and without odor if membranes ruptured is a normal indication.

Fetal factors that possibly indicate electronic fetal monitoring include which of the following? 1. Meconium passage 2. Multiple gestation 3. Preeclampsia 4. Grand multiparity 5. Decreased fetal movement

Answer: 1, 2, 5 Explanation: 1. Meconium passage is an indicator for electronic fetal monitoring. 2. Multiple gestation is an indicator for electronic fetal monitoring. 5. Decreased fetal movement is an indicator for electronic fetal monitoring.

3) A client in labor is requesting pain medication. The nurse assesses her labor status first, focusing on which of the following? Note: Credit will be given only if all correct choices and no incorrect choices are selected. Select all that apply. 1. Contraction pattern 2. Amount of cervical dilatation 3. When the labor began 4. Whether the membranes are intact or ruptured 5. Fetal presenting part

Answer: 1, 2, 5 Explanation: 1. The client should have a good contraction pattern before receiving an analgesic. 2. The nurse should evaluate the amount of cervical dilatation before analgesic medication is administered. 5. If normal parameters are absent or if nonreassuring maternal or fetal factors are present, the nurse may need to complete further assessments with the physician/CNM.

During labor, the client at 4 cm suddenly becomes short of breath, cyanotic, and hypoxic. The nurse must prepare or arrange immediately for which of the following? Note: Credit will be given only if all correct choices and no incorrect choices are selected. Select all that apply. 1. Intravenous access 2. Cesarean delivery 3. Immediate vaginal delivery 4. McRoberts maneuver 5. A crash cart

Answer: 1, 2, 5 Explanation: 1. When an amniotic fluid embolism is suspected, intravenous access is obtained as quickly as possible. 2. Shortness of breath, cyanosis, and hypoxia are symptoms of an amniotic fluid embolus, which necessitates immediate cesarean delivery. 5. The chances of a code are high, so the crash cart needs to be available.

When general anesthesia is necessary for a cesarean delivery, what should the nurse be prepared to do? Note: Credit will be given only if all correct choices and no incorrect choices are selected. Select all that apply. 1. Administer an antacid to the client. 2. Place a wedge under her thigh. 3. Apply cricoid pressure during anesthesia intubation. 4. Preoxygenate for 3-5 minutes before anesthesia. 5. Place a Foley catheter in the client's bladder.

Answer: 1, 3, 4, 5 Explanation: 1. Prophylactic antacid therapy is given to reduce the acidic content of the stomach before general anesthesia. 3. During the process of rapid induction of anesthesia, the nurse applies cricoid pressure. 4. The woman should be preoxygenated with 3 to 5 minutes of 100% oxygen. 5. Urinary retention can be treated with the placement of an indwelling Foley catheter.

A client had an epidural inserted 2 hours ago. It is functioning well, the client is stable, and labor is progressing. Which parts of the nurse's assessment have the highest priority? Note: Credit will be given only if all correct choices and no incorrect choices are selected. Select all that apply. 1. Assess blood pressure every hour. 2. Assess the pulse rate every hour. 3. Palpate the bladder. 4. Auscultate the lungs. 5. Assess the reflexes.

Answer: 1, 3 Explanation: 1. Blood pressure should be monitored every 1 to 2 minutes for the first 10 minutes and then every 5 to 15 minutes until the block wears off because hypotension is the most common side effect of epidural anesthesia. 3. Nursing care following an epidural block includes frequent assessment of the bladder to avoid bladder distention. Catheterization may be necessary, because most women are unable to void.

A client is experiencing excessive bleeding immediately after the birth of her newborn. After speeding up the IV fluids containing oxytocin, with no noticeable decrease in the bleeding, the nurse should anticipate the physician requesting which medications? Note: Credit will be given only if all correct choices and no incorrect choices are selected. Select all that apply. 1. Methergine 2. Coumadin 3. Misoprostol 4. Serotonin reuptake inhibitors (SSRIs) 5. Nonsteroidal anti-inflammatory drugs

Answer: 1, 3 Explanation: 1. Methergine is commonly used orally for postpartum hemorrhage. 3. Misoprostol is commonly used rectally for postpartum hemorrhage.

What are the primary complications of placenta accrete? Note: Credit will be given only if all correct choices and no incorrect choices are selected. Select all that apply. 1. Maternal hemorrhage 2. Insomnia 3. Failure of the placenta to separate following birth of the infant 4. Autonomic dysreflexia 5. Shoulder dystocia

Answer: 1, 3 Explanation: 1. The primary complications of placenta accreta are maternal hemorrhage and failure of the placenta to separate following birth of the infant. 3. The primary complications of placenta accreta are maternal hemorrhage and failure of the placenta to separate following birth of the infant.

Before performing Leopold maneuvers, what would the nurse do? Note: Credit will be given only if all correct choices and no incorrect choices are selected. Select all that apply. 1. Have the client empty her bladder. 2. Place the client in Trendelenburg position. 3. Have the client lie on her back with her feet on the bed and knees bent. 4. Turn the client to her left side. 5. Have the client lie flat with her ankles crossed.

Answer: 1, 3 Explanation: 1. The woman should have recently emptied her bladder before performing Leopold maneuvers. 3. The woman should lie on her back with her abdomen uncovered. To aid in relaxation of the abdominal wall, the shoulders should be raised slightly on a pillow and the knees drawn up a little.

The nurse is planning care for a client with hydramnios. For which interventions might the nurse need to prepare the client? Note: Credit will be given only if all correct choices and no incorrect choices are selected. Select all that apply. 1. Artificial rupture of the membranes 2. Amnioinfusion 3. Amniocentesis 4. Administration of prostaglandin synthesis inhibitor 5. Administration of indomethacin

Answer: 1, 3, 4, 5 Explanation: 1. Artificial rupture may be performed to remove the excessive fluid. 3. Amniocentesis may be performed to remove some excess fluid. 4. A prostaglandin synthesis inhibitor is used to treat hydramnios. 5. Indomethacin can decrease amniotic fluid by decreasing fetal urine output.

The nurse has admitted a woman with cervical insufficiency. The nurse is aware that causes of this condition include which of the following? Note: Credit will be given only if all correct choices and no incorrect choices are selected. Select all that apply. 1. Congenital factors 2. Intercourse during pregnancy 3. Infection 4. Increased uterine volume 5. Past cervical surgeries

Answer: 1, 3, 4, 5 Explanation: 1. Congenitally incompetent cervix may be found in women exposed to diethylstilbestrol (DES) or those with a bicornuate uterus. 3. Infection or trauma can cause acquired cervical incompetence. 4. Cervical insufficiency can occur in multiple-gestation pregnancies. 5. Previous elective abortion or cervical manipulation can lead to cervical insufficiency.

The nurse expects an initial weight loss for the average postpartum client to be which of the following? 1. 5 to 8 pounds 2. 10 to 12 pounds 3. 12 to 15 pounds 4. 15 to 20 pounds

Answer: 2 Explanation: 2. An initial weight loss of 10 to 12 lbs. occurs as a result of the birth of infant, placenta, and amniotic fluid.

Usually, the family is advised to arrive at the birth setting at the beginning of the active phase of labor or when which of the following occur? Note: Credit will be given only if all correct and no incorrect choices are selected. Select all that apply. 1. Rupture of membranes (ROM) 2. Increased fetal movement 3. Decreased fetal movement 4. Any vaginal bleeding 5. Regular, frequent uterine contractions (UCs)

Answer: 1, 3, 4, 5 Explanation: 1. The family is advised to arrive at the birth setting at the beginning of the active phase of labor or when the membranes rupture. 3. The family is advised to arrive at the birth setting at the beginning of the active phase of labor or when there is decreased fetal movement. 4. The family is advised to arrive at the birth setting at the beginning of the active phase of labor or when there is any vaginal bleeding. 5. The family is advised to arrive at the birth setting at the beginning of the active phase of labor or when there are regular, frequent uterine contractions.

A first-time 22-year-old single labor client, accompanied by her boyfriend, is admitted to the labor unit with ruptured membranes and mild to moderate contractions. She is determined to be 2 centimeters dilated. Which nursing diagnoses might apply during the current stage of labor? Note: Credit will be given only if all correct choices and no incorrect choices are selected. Select all that apply. 1. Fear/Anxiety related to discomfort of labor and unknown labor outcome 2. Knowledge, Deficient, related to lack of information about pushing methods 3. Pain, Acute, related to uterine contractions, cervical dilatation, and fetal descent 4. Pain, Acute, related to perineal trauma 5. Coping: Family, Compromised, related to labor process

Answer: 1, 3, 5 Explanation: 1. A Fear/Anxiety diagnosis would apply to the first stage of labor for a first-time labor client. 3. Contractions become more regular in frequency and duration, increasing discomfort and pain. 5. The woman and her boyfriend are about to undergo one of the most meaningful and stressful events in life together. Physical and psychologic resources, coping mechanisms, and support systems will all be challenged.

The nurse is admitting a client who was diagnosed with hydramnios. The client asks why she has developed this condition. The nurse should explain that hydramnios is sometimes associated with which of the following? Note: Credit will be given only if all correct choices and no incorrect choices are selected. Select all that apply. 1. Rh sensitization 2. Postmaturity syndrome 3. Renal malformation or dysfunction 4. Maternal diabetes 5. Large-for-gestational-age infants

Answer: 1, 4 Explanation: 1. Hydramnios is associated with Rh sensitization. 4. Hydramnios is associated with maternal diabetes.

A clinic nurse is preparing diagrams of pelvic shapes. Which pelvic shapes are considered least adequate for vaginal childbirth? Note: Credit will be given only for all correct choices and no incorrect choices. Select all that apply. 1. Android 2. Anthropoid 3. Gynecoid 4. Platypelloid 5. Lambdoidal suture

Answer: 1, 4 Explanation: 1. In the android and platypelloid types, the pelvic diameters are diminished. Labor is more likely to be difficult (longer) and a cesarean birth is more likely. 4. In the android and platypelloid types, the pelvic diameters are diminished. Labor is more likely to be difficult (longer) and a cesarean birth is more likely.

The nurse is teaching a prenatal class about false labor. The nurse should teach clients that false labor most likely will include which of the following? Note: Credit will be given only for all correct choices and no incorrect choices. Select all that apply. 1. Contractions that do not intensify while walking 2. An increase in the intensity and frequency of contractions 3. Progressive cervical effacement and dilatation 4. Pain in the abdomen that does not radiate 5. Contractions are at regular intervals

Answer: 1, 4 Explanation: 1. True labor contractions intensify while walking. 4. The discomfort of true labor contractions usually starts in the back and radiates around to the abdomen.

) The nurse is caring for a postpartum client who is at risk for developing early postpartum hemorrhage. What interventions would be included in the plan of care to detect this complication? Note: Credit will be given only if all correct choices and no incorrect choices are selected. Select all that apply. 1. Weigh perineal pads if the client has a slow, steady, free flow of blood from the vagina. 2. Massage the uterus every 2 hours. 3. Maintain vascular access. 4. Obtain blood specimens for hemoglobin and hematocrit. 5. Encourage the client to void if the fundus is displaced upward or to one side.

Answer: 1, 4 Explanation: 1. Weighing the perineal pads will indicate whether the client is bleeding more than anticipated. 4. The nurse reviews hemoglobin and hematocrit levels when available, and compares them to the admission baseline.

The nurse is assessing a client who has been diagnosed with an early postpartum hemorrhage. Which findings would the nurse expect? Note: Credit will be given only if all correct choices and no incorrect choices are selected. Select all that apply. 1. A boggy fundus that does not respond to massage 2. Small clots and a moderate amount of lochia rubra on the pad 3. Decreased pulse and increased blood pressure 4. Hematoma formation or bulging/shiny skin in the perineal area 5. Rise in the level of the fundus of the uterus

Answer: 1, 4, 5 Explanation: 1. A boggy fundus indicates that the uterus is not contracted and will continue to bleed. 4. Shiny or bulging skin could indicate the presence of a hematoma. 5. The uterine cavity can distend with up to 1000 mL or more of blood causing the fundus to rise.

Narcotic analgesia is administered to a laboring client at 10:00 a.m. The infant is delivered at 12:30 p.m. What would the nurse anticipate that the narcotic analgesia could do? 1. Be used in place of preoperative sedation 2. Result in neonatal respiratory depression 3. Prevent the need for anesthesia with an episiotomy 4. Enhance uterine contractions

Answer: 2 Explanation: 2. Analgesia given too late is of no value to the woman and may cause neonatal respiratory depression.

The nurse is caring for a postpartum client who is experiencing afterpains following the birth of her third child. Which comfort measure should the nurse implement to decrease her pain? Note: Credit will be given only if all correct choices and no incorrect choices are selected. Select all that apply. 1. Offer a warm water bottle for her abdomen. 2. Call the physician to report this finding. 3. Inform her that this is not normal, and she will need an oxytocic agent. 4. Administer a mild analgesic to help with breastfeeding. 5. Administer a mild analgesic at bedtime to ensure rest.

Answer: 1, 4, 5 Explanation: 1. A warm water bottle placed against the low abdomen may reduce the discomfort of afterpains. 4. The breastfeeding mother may find it helpful to take a mild analgesic agent approximately 1 hour before feeding her infant. 5. An analgesic agent such as ibuprofen is also helpful at bedtime if the afterpains interfere with the mother's rest.

A client at 32 weeks' gestation is admitted with painless vaginal bleeding. Placenta previa has been confirmed by ultrasound. What should be included in the nursing plan? Note: Credit will be given only if all correct choices and no incorrect choices are selected. Select all that apply. 1. No vaginal exams 2. Encouraging activity 3. No intravenous access until labor begins 4. Evaluating fetal heart rate with an external monitor 5. Monitoring blood loss, pain, and uterine contractility

Answer: 1, 4, 5 Explanation: 1. Expectant management of placenta previa is made by localizing the placenta via tests that require no vaginal examination. 4. Expectant management of placenta previa, when the client is at less than 37 weeks' gestation, includes evaluating FHR with an external monitor. 5. Expectant management of placenta previa, when the client is at less than 37 weeks' gestation, includes monitoring blood loss, pain, and uterine contractility.

Which factors would the nurse observe that would indicate a new mother's early attachment to the newborn? Note: Credit will be given only if all correct choices and no incorrect choices are selected. Select all that apply. 1. Face-to-face contact and eye contact 2. Failure to choose a name for the baby 3. Decreased interest in the infant's cues 4. Pointing out familial traits of the newborn 5. Displaying satisfaction with the infant's sex

Answer: 1, 4, 5 Explanation: 1. Face-to-face contact and eye contact indicates that the mother is attracted to the infant and is attending to the infant's behavior. 4. The ability to point out family traits shows that she is pleased with the baby's appearance and recognizes the infant as belonging to the family unit. 5. Showing pleasure with the infant's appearance and sex indicates bonding is occurring.

The nurse knows that the Bishop scoring system for cervical readiness includes which of the following? Note: Credit will be given only if all correct choices and no incorrect choices are selected. Select all that apply. 1. Fetal station 2. Fetal lie 3. Fetal presenting part 4. Cervical effacement 5. Cervical softness

Answer: 1, 4, 5 Explanation: 1. Fetal station is one of the components evaluated by the Bishop scoring system. 4. Cervical effacement is one of the components evaluated by the Bishop scoring system. 5. Cervical consistency is one of the components evaluated by the Bishop scoring system.

A client who is having false labor most likely would have which of the following? Note: Credit will be given only for all correct choices and no incorrect choices. Select all that apply. 1. Contractions that do not intensify while walking 2. An increase in the intensity and frequency of contractions 3. Progressive cervical effacement and dilatation 4. Pain in the abdomen that does not radiate 5. Contractions that lessen with rest and warm tub baths

Answer: 1, 4, 5 Explanation: 1. True labor contractions intensify while walking. 4. True labor results in progressive dilation, increased intensity and frequency of contractions, and pain in the back that radiates to the abdomen. 5. In true labor, contractions do not lessen with rest and warm tub baths.

The nurse is caring for a client in the transition phase of labor and notes that the fetal monitor tracing shows average short-term and long-term variability with a baseline of 142 beats per minute. What actions should the nurse take in this situation? Note: Credit will be given only if all correct choices and no incorrect choices are selected. Select all that apply. 1. Provide caring labor support. 2. Administer oxygen via face mask. 3. Change the client's position. 4. Speed up the client's intravenous. 5. Reassure the client and her partner that she is doing fine.

Answer: 1, 5 Explanation: 1. The tracing is normal, so the nurse can continue support of the labor. 5. The nurse can reassure the client at this time, as the tracing is normal.

Slowly removing some amniotic fluid is a treatment for hydramnios. What consequence can occur with the withdrawal of fluid? 1. Preterm labor 2. Prolapsed cord 3. Preeclampsia 4. Placenta previa

Answer: 2 Explanation: 2. A needle or a fetal scalp electrode is used to make a small puncture in the amniotic sac. There is a risk that the force of the fluid could make a larger hole in the amniotic sac, thus increasing the risk of a prolapsed cord.

A postpartum woman is at increased risk for developing urinary tract problems because of which of the following? 1. Decreased bladder capacity 2. Inhibited neural control of the bladder following the use of anesthetic agents 3. Increased bladder sensitivity 4. Abnormal postpartum diuresis

Answer: 2 Explanation: 2. A postpartum woman is at increased risk for developing urinary tract problems because of inhibited neural control of the bladder following the use of anesthetic agents.

During labor, the fetus was in a brow presentation, but after a prolonged labor, the fetus converted to face presentation and was delivered vaginally with forceps assist. What should the nurse explain to the parents? 1. The infant will need to be observed for meconium aspiration. 2. Facial edema and head molding will subside in a few days. 3. The infant will be given prophylactic antibiotics. 4. Breastfeeding will need to be delayed for a day or two.

Answer: 2 Explanation: 2. Any facial edema and head molding that result from the use of forceps at birth will subside in a few days.

Upon delivery of the newborn, what nursing intervention most promotes parental attachment? 1. Placing the newborn under the radiant warmer. 2. Placing the newborn on the mother's abdomen. 3. Allowing the mother a chance to rest immediately after delivery. 4. Taking the newborn to the nursery for the initial assessment.

Answer: 2 Explanation: 2. As the baby is placed on the mother's abdomen or chest, she frequently reaches out to touch and stroke her baby. When the newborn is placed in this position, the father or partner also has a very clear, close view and can also reach out to touch the baby.

The labor and delivery nurse is reviewing charts. The nurse should inform the supervisor about which client? 1. Client at 5 cm requesting labor epidural analgesia 2. Client whose cervix remains at 6 cm for 4 hours 3. Client who has developed nausea and vomiting 4. Client requesting her partner to stay with her

Answer: 2 Explanation: 2. Average cervical change in the active phase of the first stage of labor is 1.2 cm/hour; thus, this client's lack of cervical change is unexpected, and should be reported to the supervisor.

A woman in labor asks the nurse to explain the electronic fetal heart rate monitor strip. The fetal heart rate baseline is 150 with accelerations to 165, variable decelerations to 140, and moderate long-term variability. Which statement indicates that the client understands the nurse's teaching? 1. "The most important part of fetal heart monitoring is the absence of variable decelerations." 2. "The most important part of fetal heart monitoring is the presence of variability." 3. "The most important part of fetal heart monitoring is the fetal heart rate baseline." 4. "The most important part of fetal heart monitoring is the depth of decelerations."

Answer: 2 Explanation: 2. Baseline variability is a reliable indicator of fetal cardiac and neurologic function and well-being. The opposing "push-pull" balancing between the sympathetic nervous system and the parasympathetic nervous system directly affects the FHR.

The laboring client is having moderately strong contractions lasting 60 seconds every 3 minutes. The fetal head is presenting at a -2 station. The cervix is 6 cm and 100% effaced. The membranes spontaneously ruptured prior to admission, and clear fluid is leaking. Fetal heart tones are in the 140s with accelerations to 150. Which nursing action has the highest priority? 1. Encourage the husband to remain in the room. 2. Keep the client on bed rest at this time. 3. Apply an internal fetal scalp electrode. 4. Obtain a clean-catch urine specimen.

Answer: 2 Explanation: 2. Because the membranes are ruptured and the head is high in the pelvis at a -2 station, the client should be maintained on bed rest to prevent cord prolapse.

The laboring client participated in childbirth preparation classes that strongly discouraged the use of medications and intervention during labor. The client has been pushing for two hours, and is exhausted. The physician requests that a vacuum extractor be used to facilitate the birth. The client first states that she wants the birth to be normal, then allows the vacuum extraction. Following this, what should the nurse assess the client for after the birth? 1. Elation, euphoria, and talkativeness 2. A sense of failure and loss 3. Questions about whether or not to circumcise 4. Uncertainty surrounding the baby's name

Answer: 2 Explanation: 2. Clients who participate in childbirth classes that stress the normalcy of birth may feel a sense of loss or failure if an intervention is used during their labor or birth.

What is required for any women receiving oxytocin (Pitocin)? 1. CPR 2. Continuous electronic fetal monitoring 3. Administering oxygen by mask 4. Nonstress test

Answer: 2 Explanation: 2. Continuous electronic fetal monitoring (EFM) is required for any women receiving oxytocin (Pitocin).

The nurse is training a nurse new to the labor and delivery unit. They are caring for a laboring client who will have a forceps delivery. Which action or assessment finding requires intervention? 1. Regional anesthesia is administered via pudendal block. 2. The client is instructed to push between contractions. 3. Fetal heart tones are consistently between 110 and 115. 4. The client's bladder is emptied using a straight catheter.

Answer: 2 Explanation: 2. During the contraction, as the forceps are applied, the woman should avoid pushing.

A client is admitted to the labor unit with contractions 1-2 minutes apart lasting 60-90 seconds. The client is apprehensive and irritable. This client is most likely in what phase of labor? 1. Active 2. Transition 3. Latent 4. Second

Answer: 2 Explanation: 2. During transition, contractions have a frequency of 1 1/2 to 2 minutes, a duration of 60 to 90 seconds, and are strong in intensity. When the woman enters the transition phase, she may demonstrate significant anxiety.

What can be determined based on ultrasound visualization or the lack of visualization of an intertwin membrane? 1. Toxicity 2. Amnionicity 3. Variability 4. Prematurity

Answer: 2 Explanation: 2. Evidence supports the use of ultrasound for accurately determining chorionicity and amnionicity in multiple pregnancies. Determination of amnionicity is based on ultrasound visualization or the lack of visualization of an intertwin membrane.

The client at 39 weeks' gestation is undergoing a cesarean birth due to breech presentation. General anesthesia is being used. Which situation requires immediate intervention? 1. The baby's hands and feet are blue at 1 minute after birth. 2. The fetal heart rate is 70 prior to making the skin incision. 3. Clear fluid is obtained from the baby's oropharynx. 4. The neonate cries prior to delivery of the body.

Answer: 2 Explanation: 2. Fetal bradycardia occurs when the fetal heart rate falls below 110 beats/minute during a 10-minute period of continuous monitoring. When fetal bradycardia is accompanied by decreased variability, it is considered ominous and could be a sign of fetal compromise.

The physician has determined the need for forceps. The nurse should explain to the client that the use of forceps is indicated because of which of the following? 1. Her support person is exhausted 2. Premature placental separation 3. To shorten the first stage of labor 4. To prevent fetal distress

Answer: 2 Explanation: 2. Fetal conditions indicating the need for forceps include premature placental separation, prolapsed umbilical cord, and nonreassuring fetal status.

The client is in the second stage of labor. The fetal heart rate baseline is 170, with minimal variability present. The nurse performs fetal scalp stimulation. The client's partner asks why the nurse did that. What is the best response by the nurse? 1. "I stimulated the top of the fetus's head to wake him up a little." 2. "I stimulated the top of the fetus's head to try to get his heart rate to accelerate." 3. "I stimulated the top of the fetus's head to calm the fetus down before birth." 4. "I stimulated the top of the fetus's head to find out whether he is in distress."

Answer: 2 Explanation: 2. Fetal scalp stimulation is done when there is a question regarding fetal status. An acceleration indicates fetal well-being.

The nurse is calling clients at 4 weeks postpartum. Which of the following clients should be seen immediately? 1. The client who describes feeling sad all the time 2. The client who reports hearing voices talking about the baby 3. The client who states she has no appetite and wants to sleep all day 4. The client who says she needs a refill on her sertraline (Zoloft) next week

Answer: 2 Explanation: 2. Hearing voices is an indication the client is experiencing postpartum psychosis, and is the highest priority because the voices might tell her to harm her baby.

The nurse is admitting a client with possible hydramnios. When is hydramnios most likely suspected? 1. Hydramnios is most likely suspected when there is less amniotic fluid than normal for gestation. 2. Hydramnios is most likely suspected when the fundal height increases disproportionately to the gestation. 3. Hydramnios is most likely suspected when the woman has a twin gestation. 4. Hydramnios is most likely suspected when the quadruple screen comes back positive.

Answer: 2 Explanation: 2. Hydramnios should be suspected when the fundal height increases out of proportion to the gestational age.

The client has undergone an ultrasound, which estimated fetal weight at 4500 g (9 pounds 14 ounces). Which statement indicates that additional teaching is needed? 1. "Because my baby is big, I am at risk for excessive bleeding after delivery." 2. "Because my baby is big, his blood sugars could be high after he is born." 3. "Because my baby is big, my perineum could experience trauma during the birth." 4. "Because my baby is big, his shoulders could get stuck and a collarbone broken."

Answer: 2 Explanation: 2. Hypoglycemia, not hyperglycemia, is a potential complication experienced by a macrosomic fetus.

The nursing instructor is conducting a class about attachment behaviors. Which statement by a student indicates the need for further instruction? 1. "The en face position promotes bonding and attachment." 2. "Ideally, initial skin-to-skin contact occurs after the baby has been assessed and bathed." 3. "In reciprocity, the interaction of mother and infant is mutually satisfying and synchronous." 4. "The needs of the mother and of her infant are balanced during the phase of mutual regulation."

Answer: 2 Explanation: 2. Ideally, initial skin-to-skin contact is immediate. The benefits of this practice are supported by a preponderance of evidence.

The physicians/CNM opts to use a vacuum extractor for a delivery. What does the nurse understand? 1. There is little risk with vacuum extraction devices. 2. There should be further fetal descent with the first two "pop-offs." 3. Traction is applied between contractions. 4. The woman often feels increased discomfort during the procedure.

Answer: 2 Explanation: 2. If more than three "pop-offs" occur (the suction cup pops off the fetal head), the procedure should be discontinued. Page Ref: 641

If oligohydramnios occurs in the first part of pregnancy, the nurse knows that there is a danger of which of the following? 1. Major congenital anomalies 2. Fetal adhesions 3. Maternal diabetes 4. Rh sensitization

Answer: 2 Explanation: 2. If oligohydramnios occurs in the first part of pregnancy, there is a danger of fetal adhesions (one part of the fetus may adhere to another part).

A client arrives in the labor and delivery unit and describes her contractions as occurring every 10-12 minutes, lasting 30 seconds. She is smiling and very excited about the possibility of being in labor. On exam, her cervix is dilated 2 cm, 100% effaced, and -2 station. What best describes this labor? 1. Second phase 2. Latent phase 3. Active phase 4. Transition phase

Answer: 2 Explanation: 2. In the early or latent phase of the first stage of labor, contractions are usually mild. The woman feels able to cope with the discomfort. The woman is often talkative and smiling and is eager to talk about herself and answer questions.

Which of the following would be considered a clinical sign of hemorrhage? 1. Increased blood pressure 2. Increasing pulse 3. Increased urinary output 4. Hunger

Answer: 2 Explanation: 2. Increasing pulse, widening pulse pressure would be considered a clinical sign of hemorrhage.

The laboring client and her partner have arrived at the birthing unit. Which step of the admission process should be undertaken first? 1. The sterile vaginal exam 2. Welcoming the couple 3. Auscultation of the fetal heart rate 4. Checking for ruptured membranes

Answer: 2 Explanation: 2. It is important to establish rapport and to create an environment in which the family feels free to ask questions. The support and encouragement of the nurse in maintaining a caring environment begin with the initial admission.

A client calls the labor and delivery unit and tells the nurse that she is 39 weeks pregnant and that over the last 4 or 5 days, she has noticed that although her breathing has become easier, she is having leg cramps, a slight amount of edema in her lower legs, and an increased amount of vaginal secretions. The nurse tells the client that she has experienced which of the following? 1. Engagement 2. Lightening 3. Molding 4. Braxton Hicks contractions

Answer: 2 Explanation: 2. Lightening describes the effect occurring when the fetus begins to settle into the pelvic inlet.

The need for forceps has been determined. The client's cervix is dilated to 10 cm, and the fetus is at +2 station. What category of forceps application would the nurse anticipate? 1. Input 2. Low 3. Mid 4. Outlet

Answer: 2 Explanation: 2. Low forceps are applied when the leading edge of the fetal head is at +2 station.

The nurse is assisting a multiparous woman to the bathroom for the first time since her delivery 3 hours ago. When the client stands up, blood runs down her legs and pools on the floor. The client turns pale and feels weak. What would be the first action of the nurse? 1. Assist the client to empty her bladder 2. Help the client back to bed to check the fundus 3. Assess her blood pressure and pulse 4. Begin an IV of lactated Ringer's solution

Answer: 2 Explanation: 2. Massaging the fundus is the top priority because of the excessive blood loss. If the fundus is not firm, gentle fundal massage is performed until the uterus contracts.

The laboring client brought a written birth plan indicating that she wanted to avoid pain medications and an epidural. She is now at 6 cm and states, "I can't stand this anymore! I need something for pain! How will an epidural affect my baby?" What is the nurse's best response? 1. "The narcotic in the epidural will make both you and the baby sleepy." 2. "It is unlikely that an epidural will decrease your baby's heart rate." 3. "Epidurals tend to cause low blood pressure in babies after birth." 4. "I can't get you an epidural, because of your birth plan."

Answer: 2 Explanation: 2. Maternal hypotension results in uteroplacental insufficiency in the fetus, which is manifested as late decelerations on the fetal monitoring strip. The risk of hypotension can be minimized by hydrating the vascular system with 500 to 1000 mL of IV solution before the procedure and changing the woman's position and/or increasing the IV rate afterward.

The nurse is caring for a client with hydramnios. What will the nurse watch for? 1. Possible intrauterine growth restriction 2. Newborn congenital anomalies 3. Newborn postmaturity and renal malformations 4. Fetal adhesions

Answer: 2 Explanation: 2. Newborn congenital anomalies occur with hydramnios

A client received epidural anesthesia during the first stage of labor. The epidural is discontinued immediately after delivery. This client is at increased risk for which problem during the fourth stage of labor? 1. Nausea 2. Bladder distention 3. Uterine atony 4. Hypertension

Answer: 2 Explanation: 2. Nursing care following an epidural block includes frequent assessment of the bladder to avoid bladder distention.

The nurse should anticipate the labor pattern for a fetal occiput posterior position to be which of the following? 1. Shorter than average during the latent phase 2. Prolonged as regards the overall length of labor 3. Rapid during transition 4. Precipitous

Answer: 2 Explanation: 2. Occiput posterior (OP) position of the fetus is the most common fetal malposition and occurs when the head remains in the direct OP position throughout labor. This can prolong the overall length of labor.

What type of forceps are designed to be used with a breech presentation? 1. Midforceps 2. Piper 3. Low 4. High

Answer: 2 Explanation: 2. Piper forceps are designed to be used with a breech presentation. They are applied after the birth of the body, when the fetal head is still in the birth canal and assistance is needed.

The client delivered 30 minutes ago. Her blood pressure and pulse are stable. Vaginal bleeding is scant. The nurse should prepare for which procedure? 1. Abdominal hysterectomy 2. Manual removal of the placenta 3. Repair of perineal lacerations 4. Foley catheterization

Answer: 2 Explanation: 2. Retention of the placenta beyond 30 minutes after birth is termed retained placenta. Manual removal of the placenta is then performed.

A client was admitted to the labor area at 5 cm with ruptured membranes about 14 hours ago. What assessment data would be most beneficial for the nurse to collect? 1. Blood pressure 2. Temperature 3. Pulse 4. Respiration

Answer: 2 Explanation: 2. Rupture of membranes places the mother at risk for infection. The temperature is the primary and often the first indication of a problem.

Four minutes after the birth of a baby, there is a sudden gush of blood from the mother's vagina, and about 8 inches of umbilical cord slides out. What action should the nurse take first? 1. Place the client in McRoberts position. 2. Watch for the emergence of the placenta. 3. Prepare for the delivery of an undiagnosed twin. 4. Place the client in a supine position.

Answer: 2 Explanation: 2. Signs of placental separation usually appear around 5 minutes after birth of the infant, but can take up to 30 minutes to manifest. These signs are (1) a globular-shaped uterus, (2) a rise of the fundus in the abdomen, (3) a sudden gush or trickle of blood, and (4) further protrusion of the umbilical cord out of the vagina.

A woman has been admitted for an external version. She has completed an ultrasound exam and is attached to the fetal monitor. Prior to the procedure, why will terbutaline be administered? 1. To provide analgesia 2. To relax the uterus 3. To induce labor 4. To prevent hemorrhage

Answer: 2 Explanation: 2. Terbutaline is administered to achieve uterine relaxation.

The laboring client is at 7 cm, with the vertex at a +1 station. Her birth plan indicates that she and her partner took Lamaze prenatal classes, and they have planned on a natural, unmedicated birth. Her contractions are every 3 minutes and last 60 seconds. She has used relaxation and breathing techniques very successfully in her labor until the last 15 minutes. Now, during contractions, she is writhing on the bed and screaming. Her labor partner is rubbing the client's back and speaking to her quietly. Which nursing diagnosis should the nurse incorporate into the plan of care for this client? 1. Fear/Anxiety related to discomfort of labor and unknown labor outcome 2. Pain, Acute, related to uterine contractions, cervical dilatation, and fetal descent 3. Coping: Family, Compromised, related to labor process 4. Knowledge, Deficient, related to lack of information about normal labor process and comfort measures

Answer: 2 Explanation: 2. The client is exhibiting signs of acute pain, which is both common and expected in the transitional phase of labor.

Every time the nurse enters the room of a postpartum client who gave birth 3 hours ago, the client asks something else about her birth experience. What action should the nurse take? 1. Answer questions quickly and try to divert her attention to other subjects. 2. Review the documentation of the birth experience and discuss it with her. 3. Contact the physician to warn him the client might want to file a lawsuit, based on her preoccupation with the birth experience. 4. Submit a referral to Social Services because of possible obsessive behavior.

Answer: 2 Explanation: 2. The client may talk about her labor and birth experience. The nurse should provide opportunities to discuss the birth experience in a nonjudgmental atmosphere if the woman desires to do so.

The client presents for cervical ripening in anticipation of labor induction tomorrow. What should the nurse include in her plan of care for this client? 1. Apply an internal fetal monitor. 2. Monitor the client using electronic fetal monitoring. 3. Withhold oral intake and start intravenous fluids. 4. Place the client in a upright, sitting position.

Answer: 2 Explanation: 2. The client should be monitored using electronic fetal monitoring for at least 30 minutes and up to 2 hours after placement to assess the contraction pattern and the fetal status.

The nurse is admitting a client to the birthing unit. What question should the nurse ask to gain a better understanding of the client's psychosocial status? 1. "How did you decide to have your baby at this hospital?" 2. "Who will be your labor support person?" 3. "Have you chosen names for your baby yet?" 4. "What feeding method will you use for your baby?"

Answer: 2 Explanation: 2. The expectant mother's partner or support person is an important member of the birthing team, and assessments of the couple's coping, interactions, and teamwork are integral to the nurse's knowledge base. The nurse's physical presence with the laboring woman provides the best opportunity for ongoing assessment.

In succenturiate placenta, one or more accessory lobes of fetal villi have developed on the placenta, with vascular connections of fetal origin. What is the gravest maternal danger? 1. Cord prolapse 2. Postpartum hemorrhage 3. Paroxysmal hypertension 4. Brachial plexus injury

Answer: 2 Explanation: 2. The gravest maternal danger is postpartum hemorrhage if this minor lobe is severed from the placenta and remains in the uterus.

The client at 38 weeks' gestation has been diagnosed with oligohydramnios. Which statement indicates that teaching about the condition has been effective? 1. "My gestational diabetes might have caused this problem to develop." 2. "When I go into labor, I should come to the hospital right away." 3. "This problem was diagnosed with blood and urine tests." 4. "Women with this condition usually do not have a cesarean birth."

Answer: 2 Explanation: 2. The incidence of cord compression and resulting fetal distress is high when there is an inadequate amount of amniotic fluid. The client with oligohydramnios should come to the hospital in early labor.

A fetal weight is estimated at 4490 grams in a client at 38 weeks' gestation. Counseling should occur before labor regarding which of the following? 1. Mother's undiagnosed diabetes 2. Likelihood of a cesarean delivery 3. Effectiveness of epidural anesthesia with a large fetus 4. Need for early delivery

Answer: 2 Explanation: 2. The likelihood of a cesarean delivery with a fetus over 4000 grams is high. This should be discussed with the client before labor.

Dystocia encompasses many problems in labor. What is the most common? 1. Meconium-stained amniotic fluid 2. Dysfunctional uterine contractions 3. Cessation of contractions 4. Changes in the fetal heart rate

Answer: 2 Explanation: 2. The most common problem is dysfunctional (or uncoordinated) uterine contractions that result in a prolongation of labor.

The community nurse is meeting a new mother for the first time. The client delivered her first child 5 days ago after a 12-hour labor. Neither the mother nor the infant had any complications during the birth or postpartum period. Which statement by the client would indicate to the nurse that the client is experiencing postpartum blues? 1. "I am so happy and blessed to have my new baby." 2. "One minute I'm laughing and the next I'm crying." 3. "My husband is helping out by changing the baby at night." 4. "Breastfeeding is going quite well now that the engorgement is gone."

Answer: 2 Explanation: 2. The postpartum blues consist of a transient period of depression that occurs during the first few days of puerperium. Symptoms may include mood swings, anger, weepiness, anorexia, difficulty sleeping, and a feeling of letdown.

A laboring client has received an order for epidural anesthesia. In order to prevent the most common complication associated with this procedure, what would the nurse expect to do? 1. Observe fetal heart rate variability 2. Hydrate the vascular system with 500-1000 mL of intravenous fluids 3. Place the client in the semi-Fowler's position 4. Teach the client appropriate breathing techniques

Answer: 2 Explanation: 2. The risk of hypotension can be minimized by hydrating the vascular system with 500 to 1000 mL of IV solution before the procedure and changing the woman's position and/or increasing the IV rate afterward.

The neonate was born 5 minutes ago. The body is bluish. The heart rate is 150. The infant is crying strongly. The infant cries when the sole of the foot is stimulated. The arms and legs are flexed, and resist straightening. What should the nurse record as this infant's Apgar score? 1. 7 2. 8 3. 9 4. 10

Answer: 2 Explanation: 2. The strong cry earns 2 points. The crying with foot sole stimulation earns 2 points. The limb flexion and resistance earn 2 points each. Bluish color earns 0 points. The Apgar score is 8.

During the nursing assessment of a woman with ruptured membranes, the nurse suspects a prolapsed umbilical cord. What would the nurse's priority action be? 1. To help the fetal head descend faster 2. To use gravity and manipulation to relieve compression on the cord 3. To facilitate dilation of the cervix with prostaglandin gel 4. To prevent head compression

Answer: 2 Explanation: 2. The top priority is to relieve compression on the umbilical cord to allow blood flow to reach the fetus. It is because some obstetric maneuvers to relieve cord compression are complicated that cesarean birth is sometimes necessary.

The nurse has received the end-of-shift report on the postpartum unit. Which client should the nurse see first? 1. Woman who is 2nd day post-cesarean, moderate lochia serosa 2. Woman day of delivery, fundus firm 2 cm above umbilicus 3. Woman who had a cesarean section, 1st postpartum day, 4 cm diastasis recti abdominis 4. Woman who had a cesarean section, 1st postpartum day, hypoactive bowel sounds all quadrants

Answer: 2 Explanation: 2. This client is the top priority. The fundus should not be positioned above the umbilicus after delivery. If the fundus is in the midline but higher than expected, it is usually associated with clots within the uterus.

The client at 40 weeks' gestation reports to the nurse that she has had increased pelvic pressure and increased urinary frequency. Which response by the nurse is best? 1. "Unless you have pain with urination, we don't need to worry about it." 2. "These symptoms usually mean the baby's head has descended further." 3. "Come in for an appointment today and we'll check everything out." 4. "This might indicate that the baby is no longer in a head-down position."

Answer: 2 Explanation: 2. This is the best response because it most directly addresses what the client has reported.

The nurse is working with a pregnant adolescent. The client asks the nurse how the baby's condition is determined during labor. The nurse's best response is that during labor, the nurse will do which of the following? 1. Check the client's cervix by doing a pelvic exam every 2 hours. 2. Assess the fetus's heart rate with an electronic fetal monitor. 3. Look at the color and amount of bloody show that the client has. 4. Verify that the client's contractions are strong but not too close together.

Answer: 2 Explanation: 2. This statement best answers the question the client has asked.

Before applying a cord clamp, the nurse assesses the umbilical cord. The mother asks why the nurse is doing this. What should the nurse reply? 1. "I'm checking the blood vessels in the cord to see whether it has one artery and one vein." 2. "I'm checking the blood vessels in the cord to see whether it has two arteries and one vein." 3. "I'm checking the blood vessels in the cord to see whether it has two veins and one artery." 4. "I'm checking the blood vessels in the cord to see whether it has two arteries and two veins."

Answer: 2 Explanation: 2. Two arteries and one vein are present in a normal umbilical cord.

The client has been pushing for two hours, and is exhausted. The fetal head is visible between contractions. The physician informs the client that a vacuum extractor could be used to facilitate the delivery. Which statement indicates that the client needs additional information about vacuum extraction assistance? 1. "A small cup will be put onto the baby's head, and a gentle suction will be applied." 2. "I can stop pushing and just rest if the vacuum extractor is used." 3. "The baby's head might have some swelling from the vacuum cup." 4. "The vacuum will be applied for a total of ten minutes or less."

Answer: 2 Explanation: 2. Vacuum extraction is an assistive delivery. The physician/CNM applies traction in coordination with uterine contractions.

The nurse is planning an in-service educational program to talk about disseminated intravascular coagulation (DIC). The nurse should identify which conditions as risk factors for developing DIC? Note: Credit will be given only if all correct choices and no incorrect choices are selected. Select all that apply. 1. Diabetes mellitus 2. Abruptio placentae 3. Fetal demise 4. Multiparity 5. Preterm labor

Answer: 2, 3 Explanation: 2. As a result of the damage to the uterine wall and the retroplacental clotting with covert abruption, large amounts of thromboplastin are released into the maternal blood supply, which in turn triggers the development of disseminated intravascular coagulation (DIC) and the resultant hypofibrinogenemia. 3. Perinatal mortality associated with abruptio placentae is approximately 25%. If fetal hypoxia progresses unchecked, irreversible brain damage or fetal demise may result.

A client has just arrived in the birthing unit. What steps would be most important for the nurse to perform to gain an understanding of the physical status of the client and her fetus? Note: Credit will be given only if all correct choices and no incorrect choices are selected. Select all that apply. 1. Check for ruptured membranes and apply a fetal scalp electrode. 2. Auscultate the fetal heart rate between and during contractions. 3. Palpate contractions and resting uterine tone. 4. Assess the blood pressure, temperature, respiratory rate, and pulse rate. 5. Perform a vaginal exam for cervical dilation, and perform Leopold maneuvers.

Answer: 2, 3 Explanation: 2. Fetal heart rate auscultation gives information about the physical status of the fetus. 3. Contraction palpation provides information about the frequency, duration, and intensity of the contractions.

The postpartum client is suspected of having acute cystitis. Which symptoms would the nurse expect to see in this client? Note: Credit will be given only if all correct choices and no incorrect choices are selected. Select all that apply. 1. High fever 2. Frequency 3. Suprapubic pain 4. Chills 5. Nausea and vomiting

Answer: 2, 3 Explanation: 2. Frequency is characteristic of acute cystitis. 3. Suprapubic pain is characteristic of acute cystitis.

The nurse is working with a new mother who follows Muslim traditions. Which expectations and actions are appropriate for this client? Note: Credit will be given only if all correct choices and no incorrect choices are selected. Select all that apply. 1. To be sure she gets a kosher diet. 2. Expect that most visitors will be women. 3. Uncover only the necessary skin when assessing. 4. The father will take an active role in infant care. 5. She will prefer a male physician.

Answer: 2, 3 Explanation: 2. In Muslim cultures, emphasis on childrearing and infant care activities is on the mother and female relatives. 3. Women of the Islamic faith may have specific modesty requirements; the woman must be completely covered, with only her feet and hands exposed.

Under which circumstances would the nurse remove prostaglandin from the client's cervix? Note: Credit will be given only if all correct choices and no incorrect choices are selected. Select all that apply. 1. Contractions every 5 minutes 2. Nausea and vomiting 3. Uterine tachysystole 4. Cardiac tachysystole 5. Baseline fetal heart rate of 140-148

Answer: 2, 3, 4 Explanation: 2. A reason to remove prostaglandin from a client's cervix is the presence of nausea and vomiting. 3. A reason to remove prostaglandin from a client's cervix is uterine tachysystole. 4. A reason to remove prostaglandin from a client's cervix is cardiac tachysystole.

The nurse knows that which of the following are advantages of spinal block? Note: Credit will be given only if all correct answers and no incorrect answers are selected. Select all that apply. 1. Intense blockade of sympathetic fibers 2. Relative ease of administration 3. Maternal compartmentalization of the drug 4. Immediate onset of anesthesia 5. Larger drug volume

Answer: 2, 3, 4 Explanation: 2. One of advantages of spinal block is the relative ease of administration. 3. One of the advantages of spinal block is the maternal compartmentalization of the drug. 4. One of the advantages of spinal block is the immediate onset of anesthesia.

Nonreassuring fetal status often occurs with a tachysystole contraction pattern. Intrauterine resuscitation measures may become warranted and can include which of the following measures? Note: Credit will be given only if all correct choices and no incorrect choices are selected. Select all that apply. 1. Position the woman on her right side. 2. Apply oxygen via face mask. 3. Call for anesthesia provider for support. 4. Increase intravenous fluids by at least 700 mL bolus. 5. Call the physician/CNM to the bedside.

Answer: 2, 3, 4 Explanation: 2. The nurse would apply oxygen via face mask. 3. The nurse would call for anesthesia provider for support. 4. The nurse would increase intravenous fluids by at least 500 mL bolus.

The nurse knows that the maternal risks associated with postterm pregnancy include which of the following? Note: Credit will be given only if all correct choices and no incorrect choices are selected. Select all that apply. 1. Polyhydramnios 2. Maternal hemorrhage 3. Maternal anxiety 4. Forceps-assisted delivery 5. Perineal damage

Answer: 2, 3, 4, 5 Explanation: 2. Maternal symptoms and complications in postterm pregnancy may include maternal hemorrhage. 3. Maternal symptoms and complications in postterm pregnancy may include maternal anxiety. 4. Maternal symptoms and complications in postterm pregnancy may include an operative vaginal birth with forceps or vacuum extractor. 5. Maternal symptoms and complications in postterm pregnancy may include perineal trauma and damage.

Which of the following potential problems would the nurse consider when planning care for a client with a persistent occiput posterior position of the fetus? Note: Credit will be given only if all correct choices and no incorrect choices are selected. Select all that apply. 1. Increased fetal mortality 2. Severe perineal lacerations 3. Ceasing of labor progress 4. Fetus born in posterior position 5. Intense back pain during labor

Answer: 2, 3, 4, 5 Explanation: 2. The woman can have third- or fourth-degree perineal laceration or extension of a midline episiotomy. 3. Sometimes labor progress ceases if the fetus fails to rotate to an occiput anterior position. 4. Occiput posterior positions are associated with a higher incidence of vacuum-assisted births. 5. The woman usually experiences intense back pain in the small of her back throughout labor.

The nurse is aware of the different breathing techniques that are used during labor. Why are breathing techniques used during labor? Note: Credit will be given only if all correct choices and no incorrect choices are selected. Select all that apply. 1. They are a form of anesthesia. 2. They are a source of relaxation. 3. They increase the ability to cope with contractions. 4. They are a source of distraction. 5. They increase a woman's pain threshold.

Answer: 2, 3, 4, 5 Explanation: 2. When used correctly, breathing techniques can encourage relaxation. 3. When used correctly, breathing techniques can enhance the ability to cope with uterine contractions. 4. When used correctly, breathing techniques provide some distraction from the pain. 5. When used correctly, breathing techniques increase a woman's pain threshold.

Which of the following symptoms would be an indication of postpartum blues? Note: Credit will be given only if all correct choices and no incorrect choices are selected. Select all that apply. 1. Overeating 2. Anger 3. Mood swings 4. Constant sleepiness 5. Crying

Answer: 2, 3, 5 Explanation: 2. Anger would be a symptom of postpartum blues. 3. Mood swings would be a symptom of postpartum blues. 5. Weepiness and crying would be a symptom of postpartum blues.

Maternal risks of occiput posterior (OP) malposition include which of the following? Note: Credit will be given only if all correct choices and no incorrect choices are selected. Select all that apply. 1. Blood loss greater than 1000 mL 2. Postpartum infection 3. Anal sphincter injury 4. Higher rates of vaginal birth 5. Instrument delivery

Answer: 2, 3, 5 Explanation: 2. Postpartum infection is a maternal risk of OP. 3. Anal sphincter injury is a maternal risk of OP. 5. Instrument delivery is a maternal risk of OP.

Nursing interventions that foster the process of becoming a mother include which of the following? 1. Encouraging detachment from the nurse-patient relationship 2. Promoting maternal-infant attachment 3. Building awareness of and responsiveness to infant interactive capabilities 4. Instruct about promoting newborn independence 5. Preparing the woman for the maternal social role

Answer: 2, 3, 5 Explanation: 2. Promoting maternal-infant attachment is a nursing intervention that fosters the process of becoming a mother. 3. Building awareness of and responsiveness to infant interactive capabilities is a nursing intervention that fosters the process of becoming a mother. 5. Preparing the woman for the maternal social role is a nursing intervention that fosters the process of becoming a mother.

Which physical assessment findings would the nurse consider normal for the postpartum client following a vaginal delivery? Note: Credit will be given only if all correct choices and no incorrect choices are selected. Select all that apply. 1. Elevated blood pressure 2. Fundus firm and midline 3. Moderate amount of lochia serosa 4. Edema and bruising of perineum 5. Inflamed hemorrhoids

Answer: 2, 4 Explanation: 2. A firm fundus that is midline indicates the normal progression of uterine involution. 4. During the early postpartum period, the soft tissue in and around the perineum may appear edematous with some bruising.

In which clinical situations would it be appropriate for an obstetrician to order a labor nurse to perform amnioinfusion? Note: Credit will be given only if all correct choices and no incorrect choices are selected. Select all that apply. 1. Placental abruption 2. Meconium-stained fluid 3. Polyhydramnios 4. Variable decelerations 5. Early decelerations

Answer: 2, 4 Explanation: 2. The physician may order amnioinfusion for meconium-stained fluid. 4. Amnioinfusion is sometimes done to prevent the possibility of variable decelerations.

The nurse determines that a client is carrying her fetus in the vertical (longitudinal) lie. The nurse's judgment should be questioned if the fetal presenting part is which of the following? Note: Credit will be given only for all correct choices and no incorrect choices. Select all that apply. 1. Sacrum 2. Left arm 3. Mentum 4. Left scapula 5. Right scapula

Answer: 2, 4, 5 Explanation: 2. A fetus with an arm presenting is likely in a horizontal lie. 4. A fetus with a left scapula presenting is in a horizontal lie. 5. A fetus with a right scapula presenting is in a horizontal lie.

The nurse is monitoring a client who is receiving an amnioinfusion. Which assessments must the nurse perform to prevent a serious complication? Note: Credit will be given only if all correct choices and no incorrect choices are selected. Select all that apply. 1. Color of amniotic fluid 2. Maternal blood pressure 3. Cervical effacement 4. Uterine resting tone 5. Fluid leaking from the vagina

Answer: 2, 4, 5 Explanation: 2. Blood pressure should be monitored along with other vital signs. 4. The nurse should monitor contraction status (frequency, duration, intensity, resting tone, and associated maternal discomfort). 5. The nurse should continually check to make sure the infused fluid is being expelled from the vagina.

During the first several postpartum weeks, the new mother must accomplish certain physical and developmental tasks, including which of the following? Note: Credit will be given only if all correct choices and no incorrect choices are selected. Select all that apply. 1. Establish a therapeutic relationship with her physician 2. Adapt to altered lifestyles and family structure resulting from the addition of a new member 3. Restore her intellectual abilities 4. Restore physical condition 5. Develop competence in caring for and meeting the needs of her infant

Answer: 2, 4, 5 Explanation: 2. During the first several postpartum weeks, the new mother must adapt to altered lifestyles and family structure resulting from the addition of a new member. 4. During the first several postpartum weeks, the new mother must restore her physical condition. 5. During the first several postpartum weeks, the new mother must develop competence in caring for and meeting the needs of her infant.

) Risk factors for labor dystocia include which of the following? Note: Credit will be given only if all correct choices and no incorrect choices are selected. Select all that apply. 1. Tall maternal height 2. Labor induction 3. Small-for-gestational-age (SGA) fetus 4. Malpresentation 5. Prolonged latent phase

Answer: 2, 4, 5 Explanation: 2. Labor induction is a risk factor of dystocia. 4. Malpresentation is a risk factor of dystocia. 5. Prolonged latent phase is a risk factor of dystocia.

Amniotomy as a method of labor induction has which of the following advantages? Note: Credit will be given only if all correct choices and no incorrect choices are selected. Select all that apply. 1. The danger of a prolapsed cord is decreased. 2. There is usually no risk of hypertonus or rupture of the uterus. 3. The intervention can cause a decrease in pain. 4. The color and composition of amniotic fluid can be evaluated. 5. The contractions elicited are similar to those of spontaneous labor.

Answer: 2, 4, 5 Explanation: 2. There is usually no risk of hypertonus or rupture of the uterus and this is an advantage of amniotomy. 4. The color and composition of amniotic fluid can be evaluated and this is an advantage of amniotomy. 5. The contractions elicited are similar to those of spontaneous labor and this is an advantage of amniotomy.

A client at 40 weeks' gestation is to undergo stripping of the membranes. The nurse provides the client with information about the procedure. Which information is accurate? Note: Credit will be given only if all correct choices and no incorrect choices are selected. Select all that apply. 1. Intravenous administration of oxytocin will be used to initiate contractions. 2. The physician/CNM will insert a gloved finger into the cervical os and rotate the finger 360 degrees. 3. Stripping of the membranes will not cause discomfort, and is usually effective. 4. Labor should begin within 24-48 hours after the procedure. 5. Uterine contractions, cramping, and a bloody discharge can occur after the procedure.

Answer: 2, 4, 5 Explanation: 2. This motion separates the amniotic membranes that are lying against the lower uterine segment and internal os from the distal part of the lower uterine segment. 4. If labor is initiated, it typically begins within 24-48 hours. 5. Uterine contractions, cramping, scant bleeding, and bloody discharge can occur after stripping of the membranes.

An anesthesiologist informs the nurse that a client scheduled for a caesarean section will be having general anesthesia with postoperative self-controlled analgesia. For which clients would a general anesthesia be recommended? Note: Credit will be given only if all correct choices and no incorrect choices are selected. Select all that apply. 1. The client with a history of hypertension 2. The client who has had a lower back fusion 3. The client who is 13 years old 4. The client who is allergic to morphine sulfate 5. The client who has had surgery for scoliosis

Answer: 2, 5 Explanation: 2. Contraindications for epidural block include patients with previous back surgery. 5. Contraindications for epidural block include patients with previous back surgery.

The primary physician orders a narcotic analgesic for a client in labor. Which situations would lead the nurse to hold the medication? Note: Credit will be given only if all correct and no incorrect choices are selected. Select all that apply. 1. Contraction pattern every 3 minutes for 60 seconds 2. Fetal monitor tracing showing late decelerations 3. Client sleeping between contractions 4. Blood pressure 150/90 5. Blood pressure 80/42

Answer: 2, 5 Explanation: 2. Maternal hypotension results in uteroplacental insufficiency in the fetus, which is manifested as late decelerations on the fetal monitoring strip. 5. This would be a contraindication, as a narcotic can lower the blood pressure even more.

The primary care provider is performing a fetal scalp stimulation test. What result would the nurse hope to observe? Note: Credit will be given only if all correct choices and no incorrect choices are selected. Select all that apply. 1. Spontaneous fetal movement 2. Fetal heart acceleration 3. Increase in fetal heart variability 4. Resolution of late decelerations 5. Reactivity associated with the stimulation

Answer: 2, 5 Explanation: 2. The fetal heart rate should accelerate with stimulation. 5. There will be some reaction with the stimulation.

When counseling a newly pregnant client at 8 weeks' gestation of twins, the nurse teaches the woman about the need for increased caloric intake. What would the nurse tell the woman that the minimum recommended intake should be? 1. 2500 kcal and 120 grams protein 2. 3000 kcal and 150 grams protein 3. 4000 kcal and 135 grams protein 4. 5000 kcal and 190 grams protein

Answer: 3 Explanation: 3. 4000 kcal and 135 grams protein is the recommended caloric and protein intake in a twin-gestation pregnancy.

The client demonstrates understanding of the implications for future pregnancies secondary to her classic uterine incision when she states which of the following? 1. "The next time I have a baby, I can try to deliver vaginally." 2. "The risk of rupturing my uterus is too high for me to have any more babies." 3. "Every time I have a baby, I will have to have a cesarean delivery." 4. "I can only have one more baby."

Answer: 3 Explanation: 3. A classic uterine incision is made in the upper uterine segment and is associated with an increased risk of rupture in subsequent pregnancy, labor, and birth. Therefore, subsequent deliveries will be done by cesarean.

The nurse is aware that a fetus that is not in any stress would respond to a fetal scalp stimulation test by showing which change on the monitor strip? 1. Late decelerations 2. Early decelerations 3. Accelerations 4. Fetal dysrhythmia

Answer: 3 Explanation: 3. A fetus that is not experiencing stress responds to scalp stimulation with an acceleration of the FHR.

The client requires vacuum extraction assistance. To provide easier access to the fetal head, the physician cuts a mediolateral episiotomy. After delivery, the client asks the nurse to describe the episiotomy. What does the nurse respond? 1. "The episiotomy goes straight back toward your rectum." 2. "The episiotomy is from your vagina toward the urethra." 3. "The episiotomy is cut diagonally away from your vagina." 4. "The episiotomy extends from your vagina into your rectum."

Answer: 3 Explanation: 3. A mediolateral episiotomy is angled from the vaginal opening toward the buttock. It begins in the midline of the posterior fourchette and extends at a 45-degree angle downward to the right or left.

A client is admitted to the labor and delivery unit with a history of ruptured membranes for 2 hours. This is her sixth delivery; she is 40 years old, and smells of alcohol and cigarettes. What is this client at risk for? 1. Gestational diabetes 2. Placenta previa 3. Abruptio placentae 4. Placenta accreta

Answer: 3 Explanation: 3. Abruptio placentae is more frequent in pregnancies complicated by smoking, premature rupture of membranes, multiple gestation, advanced maternal age, cocaine use, chorioamnionitis, and hypertension.

A laboring client's obstetrician has suggested amniotomy as a method for inducing labor. Which assessment(s) must be made just before the amniotomy is performed? 1. Maternal temperature, BP, and pulse 2. Estimation of fetal birth weight 3. Fetal presentation, position, and station 4. Biparietal diameter

Answer: 3 Explanation: 3. Before an amniotomy is performed, the fetus is assessed for presentation, position, station, and FHR.

The client at 30 weeks' gestation is admitted with painless late vaginal bleeding. The nurse understands that expectant management includes which of the following? 1. Limiting vaginal exams to only one per 24-hour period. 2. Evaluating the fetal heart rate with an internal monitor. 3. Monitoring for blood loss, pain, and uterine contractibility. 4. Assessing blood pressure every 2 hours.

Answer: 3 Explanation: 3. Blood loss, pain, and uterine contractibility need to be assessed for client comfort and safety.

The nurse is preparing a client education handout on the differences between false labor and true labor. What information is most important for the nurse to include? 1. True labor contractions begin in the back and sweep toward the front. 2. False labor often feels like abdominal tightening, or "balling up." 3. True labor can be diagnosed only if cervical change occurs. 4. False labor contractions do not increase in intensity or duration.

Answer: 3 Explanation: 3. Cervical change is the only factor that actually distinguishes false from true labor. The contractions of true labor produce progressive dilatation and effacement of the cervix. The contractions of false labor do not produce progressive cervical effacement and dilatation.

The client delivered her second child 1 day ago. The client's temperature is 101.4° F, her pulse is 100, and her blood pressure is 110/70. Her lochia is moderate, serosanguinous, and malodorous. She is started on IV antibiotics. The nurse provides education for the client and her partner. Which statement indicates that teaching has been effective? 1. "This condition is called parametritis." 2. "Gonorrhea is the most common organism that causes this type of infection." 3. "My Beta-strep culture's being positive might have contributed to this problem." 4. "If I had walked more yesterday, this probably wouldn't have happened."

Answer: 3 Explanation: 3. Clinical findings of metritis in the initial 24 to 36 hours postpartum tend to be related to group B streptococcus (GBS).

The postpartum client is about to go home. The nurse includes which subject in the teaching plan? 1. Replacement of fluids 2. Striae 3. Diastasis of the recti muscles 4. REEDA scale

Answer: 3 Explanation: 3. Diastasis recti abdominis can be improved with exercise and abdominal muscle tone can improve significantly best taught when the mother is receptive to instruction during the postpartum assessment.

The client in early labor asks the nurse what the contractions are like as labor progresses. What would the nurse respond? 1. "In normal labor, as the uterine contractions become stronger, they usually also become less frequent." 2. "In normal labor, as the uterine contractions become stronger, they usually also become less painful." 3. "In normal labor, as the uterine contractions become stronger, they usually also become longer in duration." 4. "In normal labor, as the uterine contractions become stronger, they usually also become shorter in duration."

Answer: 3 Explanation: 3. During the active and transition phases, contractions become more frequent, are longer in duration, and increase in intensity.

Persistent early decelerations are noted. What would the nurse's first action be? 1. Turn the mother on her left side and give oxygen. 2. Check for prolapsed cord. 3. Do nothing. This is a benign pattern. 4. Prepare for immediate forceps or cesarean delivery.

Answer: 3 Explanation: 3. Early decelerations are considered benign, and do not require any intervention.

The laboring client with meconium-stained amniotic fluid asks the nurse why the fetal monitor is necessary, as she finds the belt uncomfortable. Which response by the nurse is most important? 1. "The monitor is necessary so we can see how your labor is progressing." 2. "The monitor will prevent complications from the meconium in your fluid." 3. "The monitor helps us to see how the baby is tolerating labor." 4. "The monitor can be removed, and oxygen given instead."

Answer: 3 Explanation: 3. Electronic fetal monitoring (EFM) provides a continuous tracing of the fetal heart rate (FHR), allowing characteristics of the FHR to be observed and evaluated.

A client dilated to 5 cm has just received an epidural for pain. She complains of feeling lightheaded and dizzy within 10 minutes after the procedure. Her blood pressure was 120/80 before the procedure and is now 80/52. In addition to the bolus of fluids she has been given, which medication is preferred to increase her BP? 1. Epinephrine 2. Terbutaline 3. Ephedrine 4. Epifoam

Answer: 3 Explanation: 3. Ephedrine is the medication of choice to increase maternal blood pressure.

The client with a normal pregnancy had an emergency cesarean birth under general anesthesia 2 hours ago. The client now has a respiratory rate of 30, pale blue nail beds, a pulse rate of 110, and a temperature of 102.6°F, and is complaining of chest pain. The nurse understands that the client most likely is experiencing which of the following? 1. Pulmonary embolus 2. Pneumococcal pneumonia 3. Pneumonitis 4. Gastroesophageal reflux disease

Answer: 3 Explanation: 3. Even when food and fluids have been withheld, the gastric juice produced during fasting is highly acidic and can produce chemical pneumonitis if aspirated. This pneumonitis is known as Mendelson syndrome. The signs and symptoms are chest pain, respiratory distress, cyanosis, fever, and tachycardia. Women undergoing emergency cesarean births appear to be at considerable risk for adverse events.

The client is being admitted to the birthing unit. As the nurse begins the assessment, the client's partner asks why the fetus's heart rate will be monitored. After the nurse explains, which statement by the partner indicates a need for further teaching? 1. "The fetus's heart rate will vary between 110 and 160." 2. "The heart rate is monitored to see whether the fetus is tolerating labor." 3. "By listening to the heart, we can tell the gender of the fetus." 4. "After listening to the heart rate, you will contact the midwife."

Answer: 3 Explanation: 3. Fetal heart rate is not a predictor of gender.

The client is recovering from a delivery that included a midline episiotomy. Her perineum is swollen and sore. Ten minutes after an ice pack is applied, the client asks for another. What is the best response from the nurse? 1. "I'll get you one right away." 2. "You only need to use one ice pack." 3. "You need to leave it off for at least 20 minutes and then reapply." 4. "I'll bring you an extra so that you can change it when you are ready."

Answer: 3 Explanation: 3. For optimal effect, the ice pack should be applied for 20 to 30 minutes and removed for at least 20 minutes before being reapplied.

The charge nurse is assessing several postpartum clients. Which client has the greatest risk for postpartum hemorrhage? 1. The client who was overdue and delivered vaginally 2. The client who delivered by scheduled cesarean delivery 3. The client who had oxytocin augmentation of labor 4. The client who delivered vaginally at 36 weeks

Answer: 3 Explanation: 3. Uterine atony is a cause of postpartal hemorrhage. A contributing factor to uterine atony is oxytocin augmentation of labor.

The nurse is providing preoperative teaching to a client for whom a cesarean birth under general anesthesia is scheduled for the next day. Which statement by the client indicates that she requires additional information? 1. "General anesthesia can be accomplished with inhaled gases." 2. "General anesthesia usually involves administering medication into my IV." 3. "General anesthesia will provide good pain relief after the birth." 4. "General anesthesia takes effect faster than an epidural

Answer: 3 Explanation: 3. General anesthesia provides no pain relief after birth, as regional anesthesia does.

A client's labor has progressed so rapidly that a precipitous birth is occurring. What should the nurse do? 1. Go to the nurse's station and immediately call the physician. 2. Run to the delivery room for an emergency birth pack. 3. Stay with the client and ask auxiliary personnel for assistance. 4. Hold back the infant's head forcibly until the physician arrives for the delivery.

Answer: 3 Explanation: 3. If birth is imminent, the nurse must not leave the client alone.

A nurse is caring for several postpartum clients. Which client is demonstrating a problem attaching to her newborn? 1. The client who is discussing how the baby looks like her father 2. The client who is singing softly to her baby 3. The client who continues to touch her baby with only her fingertips 4. The client who picks her baby up when the baby cries

Answer: 3 Explanation: 3. In a progression of touching activities, the mother proceeds from fingertip exploration of the newborn's extremities toward palmar contact with larger body areas and finally to enfolding the infant with the whole hand and arms. If the client continues to touch with only her fingertips, she might not be developing adequate early attachment.

A client in her second trimester is complaining of spotting. Causes for spotting in the second trimester are diagnosed primarily through the use of which of the following? 1. A nonstress test 2. A vibroacoustic stimulation test 3. An ultrasound 4. A contraction stress test

Answer: 3 Explanation: 3. Indirect diagnosis is made by localizing the placenta via tests that require no vaginal examination. The most commonly employed diagnostic test is the transabdominal ultrasound scan.

The labor and delivery nurse is preparing a prenatal class about facilitating the progress of labor. Which of the following frequent responses to pain should the nurse indicate is most likely to impede progress in labor? 1. Increased pulse 2. Elevated blood pressure 3. Muscle tension 4. Increased respirations

Answer: 3 Explanation: 3. It is important for the woman to relax each part of her body. Be alert for signs of muscle tension and tightening. Dissociative relaxation, controlled muscle relaxation, and specified breathing patterns are used to promote birth as a normal process.

Two hours after an epidural infusion has begun, a client complains of itching on her face and neck. What should the nurse do? 1. Remove the epidural catheter and apply a Band-Aid to the injection site. 2. Offer the client a cool cloth and let her know the itching is temporary. 3. Recognize that this is a common side effect, and follow protocol for administration of Benadryl. 4. Call the anesthesia care provider to re-dose the epidural catheter.

Answer: 3 Explanation: 3. Itching is a side effect of the medication used for an epidural infusion. Benadryl, an antihistamine, can be administered to manage pruritus.

The nurse is analyzing several fetal heart rate patterns. The pattern that would be of most concern to the nurse would be which of the following? 1. Moderate variability 2. Early decelerations 3. Late decelerations 4. Accelerations

Answer: 3 Explanation: 3. Late decelerations are caused by uteroplacental insufficiency. The late deceleration pattern is considered a nonreassuring sign.

The labor and delivery nurse is assigned to four clients in early labor. Which electronic fetal monitoring finding would require immediate intervention? 1. Early decelerations with each contraction 2. Variable decelerations that recover to the baseline 3. Late decelerations with minimal variability 4. Accelerations

Answer: 3 Explanation: 3. Late decelerations are considered a nonreassuring fetal heart rate (FHR) pattern, and therefore require immediate intervention.

The nurse is providing discharge teaching to a woman who delivered her first child 2 days ago. The nurse understands that additional information is needed if the client makes which statement? 1. "I should expect a lighter flow next week." 2. "The flow will increase if I am too active." 3. "My bleeding will remain red for about a month." 4. "I will be able to use a pantiliner in a day or two."

Answer: 3 Explanation: 3. Lochia rubra is dark red in color. It is present for the first 2 to 3 days postpartum. Lochia serosa is a pinkish color and it follows from about the 3rd to the 10th day.

The client vaginally delivers an infant that weighs 4750 g. Moderate shoulder dystocia occurred during the birth. During the initial assessment of this infant, what should the nurse look for? 1. Bell's palsy 2. Bradycardia 3. Erb palsy 4. Petechiae

Answer: 3 Explanation: 3. Macrosomic newborns should be inspected for cephalhematoma, Erb palsy, and fractured clavicles.

Major perineal trauma (extension to or through the anal sphincter) is more likely to occur if what type of episiotomy is performed? 1. Mediolateral 2. Episiorrhaphy 3. Midline 4. Medical

Answer: 3 Explanation: 3. Major perineal trauma is more likely to occur if a midline episiotomy is performed. The major disadvantage is that a tear of the midline incision may extend through the anal sphincter and rectum.

The client presents to the labor and delivery unit stating that her water broke 2 hours ago. Barring any abnormalities, how often would the nurse expect to take the client's temperature? 1. Every hour 2. Every 2 hours 3. Every 4 hours 4. Every shift

Answer: 3 Explanation: 3. Maternal temperature is taken every 4 hours unless it is above 37.5°C. If elevated, it is taken every hour.

After noting meconium-stained amniotic fluid and fetal heart rate decelerations, the physician diagnoses a depressed fetus. The appropriate nursing action at this time would be to do what? 1. Increase the mother's oxygen rate. 2. Turn the mother to the left lateral position. 3. Prepare the mother for a higher-risk delivery. 4. Increase the intravenous infusion rate.

Answer: 3 Explanation: 3. Meconium-stained fluid and heart rate decelerations are indications that delivery is considered higher-risk.

After nalbuphine hydrochloride (Nubain) is administered, labor progresses rapidly, and the baby is born less than 1 hour later. The baby shows signs of respiratory depression. Which medication should the nurse be prepared to administer to the newborn? 1. Fentanyl (Sublimaze) 2. Butorphanol tartrate (Stadol) 3. Naloxone (Narcan) 4. Pentobarbital (Nembutal)

Answer: 3 Explanation: 3. Narcan is useful for respiratory depression caused by nalbuphine (Nubain). Respiratory depression in the mother or fetus/newborn can be improved by the administration of naloxone (Narcan), which is a specific antagonist for this agent.

The client at 39 weeks' gestation calls the clinic and reports increased bladder pressure but easier breathing and irregular, mild contractions. She also states that she just cleaned the entire house. Which statement should the nurse make? 1. "You shouldn't work so much at this point in pregnancy." 2. "What you are describing is not commonly experienced in the last weeks." 3. "Your body may be telling you it is going into labor soon." 4. "If the bladder pressure continues, come in to the clinic tomorrow."

Answer: 3 Explanation: 3. One of the premonitory signs of labor is lightening: The fetus begins to settle into the pelvic inlet (engagement). With fetal descent, the uterus moves downward, and the fundus no longer presses on the diaphragm, which eases breathing.

The community nurse is working with a client whose only child is 8 months old. Which statement does the nurse expect the mother to make? 1. "I have a lot more time to myself than I thought I would have." 2. "My confidence level in my parenting is higher than I anticipated." 3. "I am constantly tired. I feel like I could sleep for a week." 4. "My baby likes everyone, and never fusses when she's held by a stranger."

Answer: 3 Explanation: 3. Physical fatigue often affects adjustments and functions of the new mother. The nurse can also provide information about the fatigue that a new mother experiences, strategies to promote rest and sleep at home, and the impact fatigue can have on a woman's emotions and sense of control.

Induction of labor is planned for a 31-year-old client at 39 weeks due to insulin-dependent diabetes. Which nursing action is most important? 1. Administer 100 mcg of misoprostol (Cytotec) vaginally every 2 hours. 2. Place dinoprostone (Prepidil) vaginal gel and ambulate client for 1 hour. 3. Begin Pitocin (oxytocin) 4 hours after 50 mcg misoprostol (Cytotec). 4. Prepare to induce labor after administering a tap water enema.

Answer: 3 Explanation: 3. Pitocin should not administered less than 4 hours after the last Cytotec dose.

The postpartum client states that she doesn't understand why she can't enjoy being with her baby. What would the nurse be concerned about? 1. Postpartum psychosis 2. Postpartum infection 3. Postpartum depression 4. Postpartum blues

Answer: 3 Explanation: 3. Postpartum depression can impair maternal-infant bonding and can cause developmental and cognitive delays in the child.

The postpartum client who is being discharged from the hospital experienced severe postpartum depression after her last birth. What should the nurse include in the plan of follow-up care for this client? 1. One visit from a homecare nurse, to take place in 2 days 2. Two visits from a public health nurse over the next month 3. An appointment with a mental health counselor 4. Follow-up with the obstetrician in 6 weeks

Answer: 3 Explanation: 3. Postpartum depression has a high recurrence rate. Women with a history of postpartum psychosis or depression or other risk factors may benefit from a referral to a mental health professional for counseling during pregnancy or postpartum.

A young adolescent is transferred to the labor and delivery unit from the emergency department. The client is in active labor, but did not know she was pregnant. What is the most important nursing action? 1. Determine who might be the father of the baby for paternity testing. 2. Ask the client what kind of birthing experience she would like to have. 3. Assess blood pressure and check for proteinuria. 4. Obtain a Social Services referral to discuss adoption.

Answer: 3 Explanation: 3. Preeclampsia is more common among adolescents than in young adults, and is potentially life-threatening to both mother and fetus. This assessment is the highest priority.

A postpartum client reports sharp, shooting pains in her nipple during breastfeeding and flaky, itchy skin on her breasts. Which of the following does the nurse suspect? 1. Nipple soreness 2. Engorgement 3. Mastitis 4. Letdown reflex

Answer: 3 Explanation: 3. Signs of mastitis include late-onset nipple pain, followed by shooting pain between feedings, often radiating to the chest wall. Eventually, the skin of the affected breast may become pink, flaking, and pruritic.

On the first postpartum day, the nurse teaches the client about breastfeeding. Two hours later, the mother seems to remember very little of the teaching. The nurse understands this memory lapse to be related to which of the following? 1. The taking-hold phase 2. Postpartum hemorrhage 3. The taking-in period 4. Epidural anesthesia

Answer: 3 Explanation: 3. Soon after birth during the taking-in period, the woman tends to be passive and somewhat dependent. She follows suggestions, hesitates about making decisions, and is still rather preoccupied with her needs.

A client who wishes to have an unmedicated birth is in the transition stage. She is very uncomfortable and turns frequently in the bed. Her partner has stepped out momentarily. How can the nurse be most helpful? 1. Talk to the client the entire time. 2. Turn on the television to distract the client. 3. Stand next to the bed with hands on the railing next to the client. 4. Sit silently in the room away from the bed.

Answer: 3 Explanation: 3. Standing next to the bed is supportive without being irritating. The laboring woman fears being alone during labor. The woman's anxiety may be decreased when the nurse remains with her.

At 1 minute after birth, the infant has a heart rate of 100 beats per minute, and is crying vigorously. The limbs are flexed, the trunk is pink, and the feet and hands are cyanotic. The infant cries easily when the soles of the feet are stimulated. How would the nurse document this infant's Apgar score? 1. 7 2. 8 3. 9 4. 10

Answer: 3 Explanation: 3. Two points each are scored in each of the categories of heart rate, respiratory effort, muscle tone, and reflex irritability. One point is scored in the category of skin color. The total Apgar would be 9.

An expectant father has been at the bedside of his laboring partner for more than 12 hours. An appropriate nursing intervention would be to do which of the following? 1. Insist that he leave the room for at least the next hour. 2. Tell him he is not being as effective as he was, and that he needs to let someone else take over. 3. Offer to remain with his partner while he takes a break. 4. Suggest that the client's mother might be of more help.

Answer: 3 Explanation: 3. Support persons frequently are reluctant to leave the laboring woman to take care of their own needs. The laboring woman often fears being alone during labor. Even though there is a support person available, the woman's anxiety may be decreased when the nurse remains with her while he takes a break.

The nurse suspects that a client has developed a perineal hematoma. What assessment findings would the nurse have detected to lead to this conclusion? 1. Facial petechiae 2. Large, soft hemorrhoids 3. Tense tissues with severe pain 4. Elevated temperature

Answer: 3 Explanation: 3. Tenseness of tissues that overlie the hematoma is characteristic of perineal hematomas.

A laboring client asks the nurse, "Why does the physician want to use an intrauterine pressure catheter (IUPC) during my labor?" The nurse would accurately explain that the best rationale for using an IUPC is which of the following? 1. The IUPC can be used throughout the birth process. 2. A tocodynamometer is invasive. 3. The IUPC provides more accurate data than does the tocodynamometer. 4. The tocodynamometer can be used only after the cervix is dilated 2 cm.

Answer: 3 Explanation: 3. The IUPC has several benefits over an external tocotransducer or palpation. Because the IUPC is inserted directly into the uterus, it provides near-exact pressure measurements for contraction intensity and uterine resting tone. The increased sensitivity of the IUPC allows for very accurate timing of uterine contractions (UCs).

If the physician indicates a shoulder dystocia during the delivery of a macrosomic fetus, how would the nurse assist? 1. Call a second physician to assist. 2. Prepare for an immediate cesarean delivery. 3. Assist the woman into McRoberts maneuver. 4. Utilize fundal pressure to push the fetus out.

Answer: 3 Explanation: 3. The McRoberts maneuver is thought to change the maternal pelvic angle and therefore reduce the force needed to extract the shoulders, thereby decreasing the incidence of brachial plexus stretching and clavicular fracture.

The student nurse is to perform Leopold maneuvers on a laboring client. Which assessment requires intervention by the staff nurse? 1. The client is assisted into supine position, and the position of the fetus is assessed. 2. The upper portion of the uterus is palpated, then the middle section. 3. After determining where the back is located, the cervix is assessed. 4. Following voiding, the client's abdomen is palpated from top to bottom.

Answer: 3 Explanation: 3. The cervical exam is not part of Leopold maneuvers. Abdominal palpation is the only technique used for Leopold maneuvers.

The nurse has just palpated contractions and compares the consistency to that of the forehead to estimate the firmness of the fundus. What would the intensity of these contractions be identified as? 1. Mild 2. Moderate 3. Strong 4. Weak

Answer: 3 Explanation: 3. The consistency of strong contractions is similar to that of the forehead.

The postpartum client has developed thrombophlebitis in her right leg. Which finding requires immediate intervention? 1. The client reports she had this condition after her last pregnancy. 2. The client develops pain and swelling in her left lower leg. 3. The client appears anxious, and describes pressure in her chest. 4. The client becomes upset that she cannot go home yet.

Answer: 3 Explanation: 3. The most common clinical findings of a pulmonary embolism include dyspnea, pleuritic chest pain, cough with or without hemoptysis, cyanosis, tachypnea and tachycardia, panic, syncope, or sudden hypotension and require immediate intervention.

The nurse is aware that labor and birth will most likely proceed normally when the fetus is in what position? 1. Right-acromion-dorsal-anterior 2. Right-sacrum-transverse 3. Occiput anterior 4. Posterior position

Answer: 3 Explanation: 3. The most common fetal position is occiput anterior. When this position occurs, labor and birth are likely to proceed normally.

What is the most significant cause of neonatal morbidity and mortality? 1. Amenorrhea 2. Posttraumatic stress disorder 3. Prematurity 4. Endometriosis

Answer: 3 Explanation: 3. The most significant cause of neonatal morbidity and mortality is prematurity and its associated complications such as respiratory distress syndrome, necrotizing enterocolitis, and intraventricular hemorrhage.

The postpartum nurse is caring for a client who gave birth to full-term twins earlier today. The nurse will know to assess for symptoms of which of the following? 1. Increased blood pressure 2. Hypoglycemia 3. Postpartum hemorrhage 4. Postpartum infection

Answer: 3 Explanation: 3. The nurse will assess for postpartum hemorrhage. Overstretching of uterine muscles with conditions such as multiple gestation, polyhydramnios, or a very large baby may set the stage for slower uterine involution.

The nurse determines the fundus of a postpartum client to be boggy. Initially, what should the nurse do? 1. Document the findings. 2. Catheterize the client. 3. Massage the uterine fundus until it is firm. 4. Call the physician immediately.

Answer: 3 Explanation: 3. The nurse would massage the uterine fundus until it is firm by keeping one hand in position and stabilizing the lower portion of the uterus. With one hand used to massage the fundus, the nurse would put steady pressure on the top of the now-firm fundus and to see if she was able to express any clots.

The client has been pushing for 2 hours and is exhausted. The physician is performing a vacuum extraction to assist the birth. Which finding is expected and normal? 1. The head is delivered after eight "pop-offs" during contractions. 2. A cephalohematoma is present on the fetal scalp. 3. The location of the vacuum is apparent on the fetal scalp after birth. 4. Positive pressure is applied by the vacuum extraction during contractions.

Answer: 3 Explanation: 3. The parents need to be informed that the caput (chignon) on the baby's head will disappear within 2 to 3 days.

The nurse is observing a new graduate perform a postpartum assessment. Which action requires intervention by the nurse? 1. Asking the client to void and donning clean gloves 2. Listening to bowel sounds and then asking when her last bowel movement occurred 3. Offering the patient pre-medication 2 hours before the assessment 4. Completing the assessment and explaining the results to the client

Answer: 3 Explanation: 3. The patient should be offered premedication 30-45 minutes before assessing the fundus, especially if the patient has had a cesareansection.

A client is admitted to the labor and delivery unit with contractions that are regular, are 2 minutes apart, and last 60 seconds. She reports that her labor began about 6 hours ago, and she had bloody show earlier that morning. A vaginal exam reveals a vertex presenting, with the cervix 100% effaced and 8 cm dilated. The client asks what part of labor she is in. The nurse should inform the client that she is in what phase of labor? 1. Latent phase 2. Active phase 3. Transition phase 4. Fourth stage

Answer: 3 Explanation: 3. The transition phase begins with 8 cm of dilatation, and is characterized by contractions that are closer and more intense.

A client is admitted to the labor and delivery unit with contractions that are 2 minutes apart, lasting 60 seconds. She reports that she had bloody show earlier that morning. A vaginal exam reveals that her cervix is 100 percent effaced and 8 cm dilated. The nurse knows that the client is in which phase of labor? 1. Active 2. Latent 3. Transition 4. Fourth

Answer: 3 Explanation: 3. The transition phase begins with 8 cm to 10 cm of dilatation, and contractions become more frequent, are longer in duration, and increase in intensity.

The nurse educator is describing the different kinds of abruptio placentae to a group of students, explaining that in a complete abruptio placentae, which of the following occurs? 1. Separation begins at the periphery of the placenta. 2. The placenta separates centrally and blood is trapped between the placenta and the uterine wall. 3. There is massive vaginal bleeding in the presence of almost total separation. 4. Blood passes between the fetal membranes and the uterine wall, and escapes vaginally.

Answer: 3 Explanation: 3. There is massive vaginal bleeding in the presence of almost total separation describes a complete separation of the placenta.

The nurse is caring for a client at 30 weeks' gestation who is experiencing preterm premature rupture of membranes (PPROM). Which statement indicates that the client needs additional teaching? 1. "If I were having a singleton pregnancy instead of twins, my membranes would probably not have ruptured." 2. "If I develop a urinary tract infection in my next pregnancy, I might rupture membranes early again." 3. "If I want to become pregnant again, I will have to plan on being on bed rest for the whole pregnancy." 4. "If I have aminocentesis, I might rupture the membranes again."

Answer: 3 Explanation: 3. There is no evidence that bed rest in a subsequent pregnancy decreases the risk for PPROM.

The nurse is admitting a client to the labor and delivery unit. Which aspect of the client's history requires notifying the physician? 1. Blood pressure 120/88 2. Father a carrier of sickle-cell trait 3. Dark red vaginal bleeding 4. History of domestic abuse

Answer: 3 Explanation: 3. Third-trimester bleeding is caused by either placenta previa or abruptio placentae. Dark red bleeding usually indicates abruptio placentae, which is life-threatening to both mother and fetus.

Which statement by a new mother 1 week postpartum indicates maternal role attainment? 1. "I don't think I'll ever know what I'm doing." 2. "This baby feels like a real stranger to me." 3. "It works better for me to undress the baby and to nurse in the chair rather than the bed." 4. "My sister took to mothering in no time. Why can't I?"

Answer: 3 Explanation: 3. This statement indicates a stage of maternal role attainment in which the new mother feels comfortable enough to make her own decisions about parenting.

Which relief measure would be most appropriate for a postpartum client with superficial thrombophlebitis? 1. Urge ambulation 2. Apply ice to the leg 3. Elevate the affected limb 4. Massage her calf

Answer: 3 Explanation: 3. Treatment for superficial thrombophlebitis involves application of local heat, elevation of the affected limb, and analgesic agents.

The postpartum client is concerned about mastitis because she experienced it with her last baby. Preventive measures the nurse can teach include which of the following? 1. Wearing a tight-fitting bra 2. Limiting breastfeedings 3. Frequent breastfeedings 4. Restricting fluid intake

Answer: 3 Explanation: 3. Treatment of mastitis includes frequent and complete emptying of the breasts.

A client is consulting a certified nurse-midwife because she is hoping for a vaginal birth after cesarean (VBAC) with this pregnancy. Which statement indicates that the client requires more information about VBAC? 1. "I can try a vaginal birth because my uterine incision is a low segment transverse incision." 2. "The vertical scar on my skin doesn't mean that the scar on my uterus goes in the same direction." 3. "There is about a 90% chance of giving birth vaginally after a cesarean." 4. "Because my hospital has a surgery staff on call 24 hours a day, I can try a VBAC there."

Answer: 3 Explanation: 3. Women whose previous cesarean was performed because of nonrecurring indications have been reported to have approximately a 60% to 80% chance of success with VBAC.

The nurse is preparing a class for mothers and their partners who have just recently delivered. One topic of the class is infant attachment. Which statement by a participant indicates an understanding of this concept? Note: Credit will be given only if all correct choices and no incorrect choices are selected. Select all that apply. 1. "We should avoid holding the baby too much." 2. "Looking directly into the baby's eyes might frighten him." 3. "Talking to the baby is good because he'll recognize our voices." 4. "Holding the baby so we have direct face-to-face contact is good." 5. "We should only touch the baby with our fingertips for the first month."

Answer: 3, 4 Explanation: 3. Attachment behaviors include cuddling, soothing, and calling the baby by name. 4. Attachment behaviors include holding the baby in the en face position.

Lacerations of the cervix or vagina may be present when bright red vaginal bleeding persists in the presence of a well-contracted uterus. The incidence of lacerations is higher among which of the following childbearing women? Note: Credit will be given only if all correct choices and no incorrect choices are selected. Select all that apply. 1. Over the age of 35 2. Have not had epidural block 3. Have had an episiotomy 4. Have had a forceps-assisted or vacuum-assisted birth 5. Nulliparous

Answer: 3, 4 Explanation: 3. The incidence of lacerations is higher among childbearing women who undergo an episiotomy. 4. The incidence of lacerations is higher among childbearing women who undergo forceps-assisted or vacuum-assisted birth.

The labor and birth nurse is admitting a client. The nurse's assessment includes asking the client whom she would like to have present for the labor and birth, and what the client would prefer to wear. The client's partner asks the nurse the reason for these questions. What would the nurse's best response be? Note: Credit will be given only if all correct choices and no incorrect choices are selected. Select all that apply. 1. "These questions are asked of all women. It's no big deal." 2. "I'd prefer that your partner ask me all the questions, not you." 3. "A client's preferences for her birth are important for me to understand." 4. "Many women have beliefs about childbearing that affect these choices." 5. "I'm gathering information that the nurses will use after the birth."

Answer: 3, 4 Explanation: 3. The nurse incorporates the family's expectations into the plan of care to be culturally appropriate and to facilitate the birth. 4. The nurse incorporates the family's expectations into the plan of care to be culturally appropriate and to facilitate the birth.

The nurse is caring for laboring clients. Which women are experiencing problems related to a critical factor of labor? Note: Credit will be given only for all correct choices and no incorrect choices. Select all that apply. 1. Woman at 7 cm, fetus in general flexion 2. Woman at 3 cm, fetus in longitudinal lie 3. Woman at 4 cm, fetus with transverse lie 4. Woman at 6 cm, fetus at -2 station, mild contractions 5. Woman at 5 cm, fetal presenting part is right shoulder

Answer: 3, 4, 5 Explanation: 3. A transverse lie occurs when the cephalocaudal axis of the fetal spine is at a right angle to the woman's spine and is associated with a shoulder presentation and can lead to complications in the later stages of labor. 4. Station refers to the relationship of the presenting part to an imaginary line drawn between the ischial spines of the maternal pelvis. If the presenting part is higher than the ischial spines, a negative number is assigned, noting centimeters above zero station. A -2 station is high in the pelvis. Contractions should be strong to cause fetal descent. Mild contractions will not move the baby down or open the cervix. This client is experiencing a problem between the maternal pelvis and the presenting part. 5. When the fetal shoulder is the presenting part, the fetus is in a transverse lie and the acromion process of the scapula is the landmark. This type of presentation occurs less than 1% of the time. This client is experiencing a problem between the maternal pelvis and the presenting part.

A woman is scheduled to have an external version for a breech presentation. The nurse carefully reviews the client's chart for contraindications to this procedure, including which of the following? Note: Credit will be given only if all correct choices and no incorrect choices are selected. Select all that apply. 1. Station -2 2. 38 weeks' gestation 3. Abnormal fetal heart rate and tracing 4. Previous cesarean section 5. Rupture of membranes

Answer: 3, 4, 5 Explanation: 3. An abnormal fetal heart rate or tracing would be a contraindication to performing a version. A nonreassuring FHR pattern might indicate that the fetus is already stressed and other action needs to be taken. 4. A previous cesarean is a contraindication for version. 5. Rupture of membranes is a contraindication for version because of insufficient amniotic fluid.

An abbreviated systematic physical assessment of the newborn is performed by the nurse in the birthing area to detect any abnormalities. Normal findings would include which of the following? Note: Credit will be given only if all correct choices and no incorrect choices are selected. Select all that apply. 1. Skin color: Body blue with pinkish extremities 2. Umbilical cord: two veins and one artery 3. Respiration rate of 30-60 irregular 4. Temperature of above 36.5°C (97.8°F) 5. Sole creases that involve the heel

Answer: 3, 4, 5 Explanation: 3. Normal findings would include a respiration rate of 30-60 irregular. 4. Normal findings would include temperature of above 36.5°C (97.8°F). 5. Normal findings would include sole creases that involve the heel.

True postterm pregnancies are frequently associated with placental changes that cause a decrease in the uterine-placental-fetal circulation. Complications related to alternations in placenta functioning include which of the following? Note: Credit will be given only if all correct choices and no incorrect choices are selected. Select all that apply. 1. Increased fetal oxygenation 2. Increased placental blood supply 3. Reduced nutritional supply 4. Macrosomia 5. Risk of shoulder dystocia

Answer: 3, 4, 5 Explanation: 3. Reduced nutritional supply is a complication related to alternations in placenta functioning. 4. Macrosomia is a complication related to alternations in placenta functioning. 5. Risk of shoulder dystocia is a complication

Five clients are in active labor in the labor unit. Which women should the nurse monitor carefully for the potential of uterine rupture? Note: Credit will be given only if all correct choices and no incorrect choices are selected. Select all that apply. 1. Age 15, in active labor 2. Age 22, with eclampsia 3. Age 25, last delivery by cesarean section 4. Age 32, first baby died during labor 5. Age 27, last delivery 11 months ago

Answer: 3, 5 Explanation: 3. A woman who has had a previous cesarean section is at risk for uterine rupture. 5. A woman who does not have at least 18 months between deliveries is at greater risk for uterine rupture.

The nurse admits into the labor area a client who is in preterm labor. What assessment finding would constitute a diagnosis of preterm labor? 1. Cervical effacement of 30% or more 2. Cervical change of 0.5 cm per hour 3. 2 contractions in 30 minutes 4. 8 contractions in 1 hour

Answer: 4 Explanation: 4. 8 contractions in a 60 minute period does define a diagnosis of preterm labor.

The midwife performs a vaginal exam and determines that the fetal head is at a -2 station. What does this indicate to the nurse about the birth? 1. The birth is imminent. 2. The birth is likely to occur in 1-2 hours. 3. The birth will occur later in the shift. 4. The birth is difficult to predict.

Answer: 4 Explanation: 4. A -2 station means that the fetus is 2 cm above the ischial spines. The ischial spines as a landmark have been designated as zero station. If the presenting part is higher than the ischial spines, a negative number is assigned, noting centimeters above zero station. With the fetus's head that high in the pelvis, it is difficult to predict when birth will occur.

A postpartum client with endometritis is being discharged home on antibiotic therapy. The new mother plans to breastfeed her baby. What should the nurse's discharge instruction include? 1. The client can douche every other day. 2. Sexual intercourse can be resumed when the client feels up to it. 3. Light housework will provide needed exercise. 4. The baby's mouth should be examined for thrush.

Answer: 4 Explanation: 4. A breastfeeding mother on antibiotics should check her baby's mouth for signs of thrush, which should be reported to the physician.

The nurse is explaining induction of labor to a client. The client asks what the indications for labor induction are. Which of the following should the nurse include when answering the client? 1. Suspected placenta previa 2. Breech presentation 3. Prolapsed umbilical cord 4. Hypertension

Answer: 4 Explanation: 4. A client with hypertension is appropriate for labor induction.

The nurse is performing a postpartum assessment on a newly delivered client. When checking the fundus, there is a gush of blood. The client asks why that is happening. What is the nurse's best response? 1. "We see this from time to time. It's not a big deal." 2. "The gush is an indication that your fundus isn't contracting." 3. "Don't worry. I'll make sure everything is fine." 4. "Blood pooled in the vagina while you were in bed."

Answer: 4 Explanation: 4. A gush of blood when a fundal massage is undertaken may occur because of normal pooling of blood in vagina when the woman lies down to rest or sleep.

The client has delivered a 4200 g fetus. The physician performed a midline episiotomy, which extended into a third-degree laceration. The client asks the nurse where she tore. Which response is best? 1. "The episiotomy extended and tore through your rectal mucosa." 2. "The episiotomy extended and tore up near your vaginal mucous membrane." 3. "The episiotomy extended and tore into the muscle layer." 4. "The episiotomy extended and tore through your anal sphincter."

Answer: 4 Explanation: 4. A third degree laceration includes the anal sphincter.

After inserting prostaglandin gel for cervical ripening, what should the nurse do? 1. Apply an internal fetal monitor. 2. Insert an indwelling catheter. 3. Withhold oral intake and start intravenous fluids. 4. Place the client in a supine position with a right hip wedge.

Answer: 4 Explanation: 4. After the gel, intravaginal insert, or tablet is inserted, the woman is instructed to remain lying down with a rolled blanket or hip wedge under her right hip to tip the uterus slightly to the left for the first 30 to 60 minutes to maintain the cervical ripening agent in place.

The nurse is preparing to assess a laboring client who has just arrived in the labor and birth unit. Which statement by the client indicates that additional education is needed? 1. "You are going to do a vaginal exam to see how dilated my cervix is." 2. "The reason for a pelvic exam is to determine how low in the pelvis my baby is." 3. "When you check my cervix, you will find out how thinned out it is." 4. "After you assess my pelvis, you will be able to tell when I will deliver."

Answer: 4 Explanation: 4. An experienced labor and birth nurse can estimate the time of delivery based on the cervix, fetal position, station, and contraction pattern. However, during a pelvic exam, no information is obtained about contractions. The nurse will not have enough information following the cervical exam to estimate time of birth.

In the operating room, a client is being prepped for a cesarean delivery. The doctor is present. What is the last assessment the nurse should make just before the client is draped for surgery? 1. Maternal temperature 2. Maternal urine output 3. Vaginal exam 4. Fetal heart tones

Answer: 4 Explanation: 4. Ascertain fetal heart rate (FHR) before surgery and during preparation because fetal hypoxia can result from aortocaval compression.

Toward the end of the first stage of labor, a pudendal block is administered transvaginally. What will the nurse anticipate the client's care will include? 1. Monitoring for hypotension every 15 minutes 2. Monitoring FHR every 15 minutes 3. Monitoring for bladder distention 4. No additional assessments

Answer: 4 Explanation: 4. Because a pudendal block is done using a local anesthetic, there is no need for additional monitoring of the mother or the fetus.

As compared with admission considerations for an adult woman in labor, the nurse's priority for an adolescent in labor would be which of the following? 1. Cultural background 2. Plans for keeping the infant 3. Support persons 4. Developmental level

Answer: 4 Explanation: 4. Because her cognitive development is incomplete, the younger adolescent may have fewer problem-solving capabilities. The very young woman needs someone to rely on at all times during labor. She may be more childlike and dependent than older teens.

The nurse is caring for a laboring client. A cervical exam indicates 8 cm dilation. The client is restless, frequently changing position in an attempt to get comfortable. Which nursing action is most important? 1. Leave the client alone so she can rest. 2. Ask the family to take a coffee-and-snack break. 3. Encourage the client to have an epidural for pain. 4. Reassure the client that she will not be left alone.

Answer: 4 Explanation: 4. Because the client is in the transitional phase of the first stage of labor, she will not want to be left alone; staying with the client and reassuring her that she will not be alone are the highest priorities at this time.

The nurse is caring for a client in active labor. The membranes spontaneously rupture, with a large amount of clear amniotic fluid. Which nursing action is most important to undertake at this time? 1. Assess the odor of the amniotic fluid. 2. Perform Leopold maneuvers. 3. Obtain an order for pain medication. 4. Complete a sterile vaginal exam.

Answer: 4 Explanation: 4. Checking the cervix will determine whether the cord prolapsed when the membranes ruptured. The nurse would assess for prolapsed cord via vaginal examination.

The client delivered her first child vaginally 7 hours ago. She has not voided since delivery. She has an IV of lactated Ringer's solution running at 100 mL/hr. Her fundus is firm and to the right of midline. What is the best nursing action? 1. To massage the fundus vigorously 2. To assess the client's pain level 3. To increase the rate of the IV 4. To assist the client to the bathroom

Answer: 4 Explanation: 4. Emptying the bladder is the top priority.

A client is having contractions that last 20-30 seconds and that are occurring every 8-20 minutes. The client is requesting something to help relieve the discomfort of contractions. What should the nurse suggest? 1. That a mild analgesic be administered 2. An epidural 3. A local anesthetic block 4. Nonpharmacologic methods of pain relief

Answer: 4 Explanation: 4. For this pattern of labor, nonpharmacologic methods of pain relief should be suggested. These can include back rubs, providing encouragement, and clean linens.

The nurse is beginning the postpartum teaching of a mother who has given birth to her first child. What aspect of teaching is most important? 1. Describe the likely reaction of siblings to the new baby. 2. Discuss adaptation to grandparenthood by her parents. 3. Determine whether father-infant attachment is taking place. 4. Assist the mother in identifying the baby's behavior cues.

Answer: 4 Explanation: 4. Helping the mother to identify her baby's behavior cues facilitates the acquaintance phase of maternal-infant attachment.

The nurse examines the client's placenta and finds that the umbilical cord is inserted at the placental margin. The client comments that the placenta and cord look different than they did for her first two births. The nurse should explain that this variation in placenta and cord is called what? 1. Placenta accreta 2. Circumvallate placenta 3. Succenturiate placenta 4. Battledore placenta

Answer: 4 Explanation: 4. In battledore placenta, the umbilical cord is inserted at or near the placental margin.

The nurse has presented a session on pain relief options to a prenatal class. Which statement indicates that additional teaching is needed? 1. "An epidural can be continuous or can be given in one dose." 2. "A spinal is usually used for a cesarean birth." 3. "Pudendal blocks are effective when a vacuum is needed." 4. "Local anesthetics provide good labor pain relief."

Answer: 4 Explanation: 4. Local anesthetics are not used for labor pain relief. They are used prior to episiotomy and for laceration repair.

The client is at 42 weeks' gestation. Which order should the nurse question? 1. Obtain biophysical profile today. 2. Begin nonstress test now. 3. Schedule labor induction for tomorrow. 4. Have the client return to the clinic in 1 week.

Answer: 4 Explanation: 4. Many practitioners use twice-weekly testing providing the amniotic fluid level is normal. One week is too long a period between assessments.

The client has stated that she wants to avoid an epidural and would like an unmedicated birth. Which nursing action is most important for this client? 1. Encourage the client to vocalize during contractions. 2. Perform vaginal exams only between contractions. 3. Provide a CD of soft music with sounds of nature. 4. Offer to teach the partner how to massage tense muscles.

Answer: 4 Explanation: 4. Massage is helpful for many clients, especially during latent and active labor. Massage can increase relaxation and therefore decrease tension and pain.

A woman is in labor. The fetus is in vertex position. When the client's membranes rupture, the nurse sees that the amniotic fluid is meconium-stained. What should the nurse do immediately? 1. Change the client's position in bed. 2. Notify the physician that birth is imminent. 3. Administer oxygen at 2 liters per minute. 4. Begin continuous fetal heart rate monitoring.

Answer: 4 Explanation: 4. Meconium-stained amniotic fluid is an abnormal fetal finding, and is an indication for continuous fetal monitoring.

The client is carrying monochorionic-monoamniotic twins. The nurse teaches the client what this is, and the implications of this finding. The nurse knows that teaching is successful when the client states which of the following? 1. "My babies came from two eggs." 2. "About two thirds of twins have this amniotic sac formation." 3. "My use of a fertility drug led to this issue." 4. "My babies have a lower chance of surviving to term than fraternal twins do."

Answer: 4 Explanation: 4. Monochorionic-monoamniotic twins are both in one amniotic sac. There is an increased risk of umbilical cords becoming tangled or knotted and a higher incidence of fetal demise.

The nurse assesses the postpartum client who has not had a bowel movement by the third postpartum day. Which nursing intervention would be appropriate? 1. Encourage the new mother, saying, "It will happen soon." 2. Instruct the client to eat a low-fiber diet. 3. Decrease fluid intake. 4. Obtain an order for a stool softener.

Answer: 4 Explanation: 4. Obtaining an order for a stool softener is the correct intervention by the third day. In resisting or delaying the bowel movement, the woman may cause increased constipation and more pain when elimination finally occurs.

) The nurse is caring for a client who recently emigrated from a Southeast Asian country. The mother has been resting since the birth, while her sister has changed the diapers and fed the infant. What is the most likely explanation for this behavior? 1. The client is not attaching to her infant appropriately. 2. The client is not going to be a good mother, and the baby is at risk. 3. The client has no mother present to role-model behaviors. 4. The client is exhibiting normal behavior for her culture.

Answer: 4 Explanation: 4. Rest, seclusion, and dietary restraint practices in many traditional non-Western cultures (South Asian groups) are designed to assist the woman and her baby during postpartum vulnerable periods.

When comparing the anterior and posterior fontanelles of a newborn, the nurse knows that both are what? 1. Both are approximately the same size 2. Both close within 12 months of birth 3. Both are used in labor to identify station 4. Both allow for assessing the status of the newborn after birth

Answer: 4 Explanation: 4. The anterior and posterior fontanelles are clinically useful in identifying the position of the fetal head in the pelvis and in assessing the status of the newborn after birth.

The charge nurse has received the shift change report. Which client requires immediate intervention? 1. Woman at 6 cm undergoing induction of labor, strong contractions every 3 minutes 2. Woman at 4 cm whose fetus is in a longitudinal lie with a cephalic presentation 3. Woman at 10 cm and fetus at +2 station experiencing a strong expulsion urge 4. Woman at 3 cm screaming in fear because her mother died during childbirth

Answer: 4 Explanation: 4. This client is most likely fearful that she will die during labor because her mother died during childbirth. This client requires education and a great deal of support, and is therefore the top priority.

During the fourth stage of labor, the client's assessment includes a BP of 110/60, pulse 90, and the fundus is firm midline and halfway between the symphysis pubis and the umbilicus. What is the priority action of the nurse? 1. Turn the client onto her left side. 2. Place the bed in Trendelenburg position. 3. Massage the fundus. 4. Continue to monitor.

Answer: 4 Explanation: 4. The client's assessment data are normal for the fourth stage of labor, so monitoring is the only action necessary. During the fourth stage of labor, the mother experiences a moderate drop in both systolic and diastolic blood pressure, increased pulse pressure, and moderate tachycardia.

During a postpartum examination of a client who delivered an 8-pound newborn 6 hours ago, the following assessment findings are noted: fundus firm and at the umbilicus, and moderate lochia rubra with a steady trickle of blood from the vagina. What is the assessment finding that would necessitate follow-up? 1. Firm fundus 2. Fundus at the umbilical level 3. Moderate lochia rubra 4. Steady trickle of blood

Answer: 4 Explanation: 4. The continuous seepage of blood is more consistent with cervical or vaginal lacerations. Lacerations should be suspected if the uterus is firm and of expected size and if no clots can be expressed. This finding would indicate a follow-up.

During a maternal assessment, the nurse determines the fetus to be in a left occiput anterior (LOA) position. Auscultation of the fetal heart rate should begin in what quadrant? 1. Right upper quadrant 2. Left upper quadrant 3. Right lower quadrant 4. Left lower quadrant

Answer: 4 Explanation: 4. The fetal heart rate (FHR) is heard most clearly at the fetal back. Thus, in a cephalic presentation, the FHR is best heard in the lower quadrant of the maternal abdomen.

While caring for a client in labor, the nurse notices during a vaginal exam that the fetus's head has rotated internally. What would the nurse expect the next set of cardinal movements for a fetus in a vertex presentation to be? 1. Flexion, extension, restitution, external rotation, and expulsion 2. Expulsion, external rotation, and restitution 3. Restitution, flexion, external rotation, and expulsion 4. Extension, restitution, external rotation, and expulsion

Answer: 4 Explanation: 4. The fetus changes position in the following order: descent, flexion, internal rotation, extension, restitution, external rotation, and expulsion.

A client has just been admitted for labor and delivery. She is having mild contractions lasting 30 seconds every 15 minutes. The client wants to have a medication-free birth. When discussing medication alternatives, the nurse should be sure the client understands which of the following? 1. In order to respect her wishes, no medication will be given. 2. Pain relief will allow a more enjoyable birth experience. 3. The use of medications allows the client to rest and be less fatigued. 4. Maternal pain and stress can have a more adverse effect on the fetus than would a small amount of analgesia.

Answer: 4 Explanation: 4. The nurse can explain to the couple that, although pharmacologic agents do affect the fetus, so do the pain and stress experienced by the laboring mother. If the woman's pain and anxiety are more than she can cope with, the adverse physiologic effects on the fetus may be as great as would occur with the administration of a small amount of an analgesic agent. Once the effects are explained, however, it is still the client's choice whether to receive medication.

After delivery, it is determined that there is a placenta accreta. Which intervention should the nurse anticipate? 1. 2 L oxygen by mask 2. Intravenous antibiotics 3. Intravenous oxytocin 4. Hysterectomy

Answer: 4 Explanation: 4. The primary complication of placenta accreta is maternal hemorrhage and failure of the placenta to separate following birth of the infant. An abdominal hysterectomy may be the necessary treatment, depending on the amount and depth of involvement.

To assess the healing of the uterus at the placental site, what does the nurse assess? 1. Lab values 2. Blood pressure 3. Uterine size 4. Type, amount, and consistency of lochia

Answer: 4 Explanation: 4. The type, amount, and consistency of lochia determine the state of healing of the placental site, and a progressive color change from bright red at birth to dark red to pink to white or clear should be observed.

Prior to receiving lumbar epidural anesthesia, the nurse would anticipate placing the laboring client in which position? 1. On her right side in the center of the bed with her back curved 2. Lying prone with a pillow under her chest 3. On her left side with the bottom leg straight and the top leg slightly flexed 4. Sitting on the edge of the bed

Answer: 4 Explanation: 4. The woman is positioned on her left or right side, at the edge of the bed with the assistance of the nurse, with her legs slightly flexed, or she is asked to sit on the edge of the bed.

Why is it important for the nurse to assess the bladder regularly and encourage the laboring client to void frequently? 1. A full bladder impedes oxygen flow to the fetus. 2. Frequent voiding prevents bruising of the bladder. 3. Frequent voiding encourages sphincter control. 4. A full bladder can impede fetal descent.

Answer: 4 Explanation: 4. The woman should be encouraged to void because a full bladder can interfere with fetal descent. If the woman is unable to void, catheterization may be necessary.

A client had a cesarean birth 3 days ago. She has tenderness, localized heat, and redness of the left leg. She is afebrile. As a result of these symptoms, what would the nurse anticipate would be the next course of action? 1. That the client would be encouraged to ambulate freely 2. That the client would be given aspirin 650 mg by mouth 3. That the client would be given Methergine IM 4. That the client would be placed on bed rest

Answer: 4 Explanation: 4. These symptoms indicate the presence of superficial thrombophlebitis. The treatment involves bed rest, elevation of the affected limb, analgesics, and use of elastic support hose.

The multiparous client at term has arrived to the labor and delivery unit in active labor with intact membranes. Leopold maneuvers indicate the fetus is in a transverse lie with a shoulder presentation. Which physician order is most important? 1. Artificially rupture membranes. 2. Apply internal fetal scalp electrode. 3. Monitor maternal blood pressure every 15 minutes. 4. Alert surgical team of urgent cesarean.

Answer: 4 Explanation: 4. This is the highest priority because vaginal birth is impossible with a transverse lie. Labor should not be allowed to continue, and a cesarean birth is done quickly.

How does the nurse assess for Homans' sign? 1. Extending the foot and inquiring about calf pain. 2. Extending the leg and inquiring about foot pain. 3. Flexing the knee and inquiring about thigh pain. 4. Dorsiflexing the foot and inquiring about calf pain.

Answer: 4 Explanation: 4. To assess for thrombophlebitis, the nurse should have the woman stretch her legs out, with the knees slightly flexed and the legs relaxed. The nurse then grasps the foot and dorsiflexes it sharply. If pain is elicited, the nurse notifies the physician/CNM that the woman has a positive Homans' sign. The pain is caused by inflammation of a vessel.

The nurse is orienting a new graduate nurse to the labor and birth unit. Which statement indicates that teaching has been effective? 1. "When a client arrives in labor, a urine specimen is obtained by catheter to check for protein and ketones." 2. "When a client arrives in labor, she will be positioned supine to facilitate a normal blood pressure." 3. "When a client arrives in labor, her prenatal record is reviewed for indications of domestic abuse." 4. "When a client arrives in labor, a vaginal exam is performed unless birth appears to be imminent."

Answer: 4 Explanation: 4. Unless delivery seems imminent because the client is bearing down or contractions are very close and strong, the vaginal exam is performed after the vital signs are obtained.

After several hours of labor, the electronic fetal monitor (EFM) shows repetitive variable decelerations in the fetal heart rate. The nurse would interpret the decelerations to be consistent with which of the following? 1. Breech presentation 2. Uteroplacental insufficiency 3. Compression of the fetal head 4. Umbilical cord compression

Answer: 4 Explanation: 4. Variable decelerations occur when there is umbilical cord compression.

The client delivered her second child yesterday, and is preparing to be discharged. She expresses concern to the nurse because she developed an upper urinary tract infection (UTI) after the birth of her first child. Which statement indicates that the client needs additional teaching about this issue? 1. "If I start to have burning with urination, I need to call the doctor." 2. "Drinking 8 glasses of water each day will help prevent another UTI." 3. "I will remember to wipe from front to back after I move my bowels." 4. "Voiding 2 or 3 times per day will help prevent a recurrence."

Answer: 4 Explanation: 4. Voiding only 2 or 3 times per day is not sufficient to prevent recurrence of a urinary tract infection (UTI). The woman needs to empty her bladder whenever she feels the urge to void at least every 2 to 4 hours while awake.

The client is having fetal heart rate decelerations. An amnioinfusion has been ordered for the client to alleviate the decelerations. The nurse understands that the type of decelerations that will be alleviated by amnioinfusion is which of the following? 1. Early decelerations 2. Moderate decelerations 3. Late decelerations 4. Variable decelerations

Answer: 4 Explanation: 4. When cord compression is suspected, amnioinfusion (AI) may be considered. AI helps to prevent the possibility of variable decelerations by increasing the volume of amniotic fluid.

Which of the following is a risk factor for urinary retention after childbirth? 1. Multiparity 2. Precipitous labor 3. Unassisted childbirth 4. Not sufficiently recovering from the effects of anesthesia

Answer: 4 Explanation: 4. Women who have not sufficiently recovered from the effects of anesthesia and cannot void spontaneously are at risk for urinary retention after childbirth.

During the initial intrapartal assessment of a client in early labor, the nurse performs a vaginal examination. The client's partner asks why this pelvic exam needs to be done. The nurse should explain that the purpose of the vaginal exam is to obtain information about which of the following? Note: Credit will be given only if all correct choices and no incorrect choices are selected. Select all that apply. 1. Uterine contraction pattern 2. Fetal position 3. Presence of the mucous plug 4. Cervical dilation and effacement 5. Presenting part

Answer: 4, 5 Explanation: 4. The vaginal examination of a laboring client obtains information about the station of the presenting part and the dilation and effacement of the cervix. 5. The vaginal examination of a laboring client obtains information about the fetal presenting part.


Conjuntos de estudio relacionados

Physiology: Appendicular Skeleton

View Set

NCLEX AQ Client Needs: Psychosocial Integrity

View Set

ECON 101 Final Exam UMass Boston- Millman

View Set

ECON 102 Chapter 11 Practice Questions

View Set

Causes of Texas Revolution Ch.9 Study Guide

View Set

Latijn woordjes SPQR tekst 17 & 18

View Set